Sunteți pe pagina 1din 67

1

Hair and Nail disorders


Which of the following syndromes does not have Dorsal pterygium as a feature:
1

Lichen planus
2

Lesch-Nyhan syndrome
3

Cicatricial pemphigoid
4

Chronic GVHD
5

Lamellar ichthyosis
Q/Q(M)-478121 Report a Problem


Which of the following syndromes does not have Dorsal pterygium as a feature:
5

Lamellar ichthyosis
Dorsal pterygium (scarring of the proximal nail fold) is caused by lichen planus, acrosclerosis,
onychotillomania, Lesch-Nyhan syndrome, chronic GVHD, SJS/TEN and cicatricial pemphigoid. All of
the listed items are correct except lamellar ichthyosis. It does not have this nail finding.
Q/Q(M)-478121 Report a Problem

The best test for Cushing syndrome is:
1

Plasma LH
2

Plasma testosterone
3

Plasma prolactin
4

Plasma DHEA
5

Overnight dexamethasone suppresion test
Q/Q(M)-478086 Report a Problem

The best test for Cushing syndrome is:
5

Overnight dexamethasone suppresion test
The overnight dexamethasone suppression test is the test of choice for diagnosis of Cushing syndrome. In
this test, a dose of dexamethasone is given at 11pm and the plasma cortisol level is drawn the following
morning. If the Cushing syndrome is from an adrenal tumor or an ectopic ACTH producing tumor, there is
no change in cortisol secretion. A normal response is suppression of the cortisol level by the extra
dexamethasone. If the Cushings is from a pituitary tumor, at low doses of dexamethasone there are no
changes in cortisol levels while at high doses of dexamethasone there will be a normal suppression. The
other listed items are not used for diagnosis of Cushing syndrome.
Q/Q(M)-478086 Report a Problem

A 34-year-old man presents with a jet-black nail plate involving the right index finger. Hutchinsons sign
is negative and the patient denies antecedent trauma. What is the most likely etiology of the nail plate
discoloration:
1

Multiple Myeloma
2

Wilsons disease
2

3

Congenital Oncychodysplasia of the Index Figner (COIF)
4

Pseudomonas infection
5

Proteus Infection
Q/Q(M)-482925 Report a Problem
A 34-year-old man presents with a jet-black nail plate involving the right index finger. Hutchinsons sign
is negative and the patient denies antecedent trauma. What is the most likely etiology of the nail plate
discoloration:
5

Proteus Infection
Proteus infection of the nail is associated with black discoloration of the nail plate. Multiple myeloma may
be associated with anonychia. Wilsons disease classically exhibits blue lunulae. COIF presents with
absence of the nail plate. Pseudomonal infections of the subunguium may produce a green discoloration
due to production of pyocyanin pigment.
Q/Q(M)-482925 Report a Problem


An 18 year-old girl who was hospitalized last month after a serious car accident is noted to have white
transverse grooves on her fingernails and toenails. The most likely diagnosis is:
1

Twenty-nail dystrophy
2

Beaus lines
3

Half-and-half nails
4

Mees lines
5

Terrys nails
Q/Q(M)-477335 Report a Problem


An 18 year-old girl who was hospitalized last month after a serious car accident is noted to have white
transverse grooves on her fingernails and toenails. The most likely diagnosis is:
2

Beaus lines
Beaus lines are transverse grooves in the nails. They are caused by a generalized systemic condition
which disrupts nail formation. Conditions may include infection, myocardial infection, neurologic events
and cytotoxic medications.
Q/Q(M)-477335 Report a Problem

Triangular lunulae are seen in what disorder?
1

Dyskeratosis congentia
2

Neurofibromatosis II
3

Gorlins syndrome
4

Nail-patella syndrome
5

Papillon-Lefevre syndrome
Q/Q(M)-474578 Report a Problem
3


Triangular lunulae are seen in what disorder?
4

Nail-patella syndrome
Triangular lunulae are seen in nail-patella syndrome which is characterized by mutation in LMX1b.
Q/Q(M)-474578 Report a Problem


Blue lunulae are characteristic of which disease?
1

Yellow nail syndrome
2

Wilson's disease
3

Rubenstein-Taybi syndrome
4

Hypertrophic pulmonary osteoarthropathy
5

Clubbing
Q/Q(M)-474576 Report a Problem


Blue lunulae are characteristic of which disease?
2

Wilson's disease
Wilson's disease is associated with characteristic blue lunulae.
Q/Q(M)-474576 Report a Problem

A patient with diffuse severe sudden hair loss developing 3 months after hospitalization for septicemia
likely has which of the following conditions?
1

Anagen effluvium
2

Catagen effluvium
3

Telogen effluvium
4

Loose anagen syndrome
5

Uncombable hair syndrome
Q/Q(M)-474567 Report a Problem


A patient with diffuse severe sudden hair loss developing 3 months after hospitalization for septicemia
likely has which of the following conditions?
3

Telogen effluvium
Telogen effluvium typically ensues stressful events after a 2-3 month period.
Q/Q(M)-474567 Report a Problem

How long should the last wash-out period before performing mycologic investigations in a clinically
recurrent onychomycosis treated by terbinafine, itraconazole or fluconazole:
1

One month
4

2

Two months
3

Three months
4

Four months
5

Six months
Q/Q(M)-475900 Report a Problem



How long should the last wash-out period before performing mycologic investigations in a clinically
recurrent onychomycosis treated by terbinafine, itraconazole or fluconazole:
3

Three months
This is an interesting point, because we know that these systemic antifungals are retained in the keratin of
the toenails up to six months after the treatment has been stopped. Therefore this answer could have been
debatable were it not for the presence of fungi in the nail bed (which is the primary location of the disease)
and where the scraping should be properly performed for mycologic testing after removal of a piece of
nail as proximal as possible.
Q/Q(M)-475900 Report a Problem

Which genodermatosis characterized by broad thumbs and this nail disorder pictured below?
1

Rothmund-Thompson
2

Tuberous sclerosis
3

Pachyonychia congenita
4

Proteus syndrome
5

Rubenstein-Taybi syndrome
Q/Q(M)-475936 Report a Problem


Which genodermatosis characterized by broad thumbs and this nail disorder pictured below?
5

Rubenstein-Taybi syndrome
Rubensteom-Taybi syndrome is characterized by broad thumbs and broad nails or brachyonychia. This
syndrome is characterized by mutations in CREB-binding protein.
Q/Q(M)-475936 Report a Problem
5



The combination of poliosis, uveitis, deafness, and vitiligo are characteristic of which of the following
conditions?
1

Piebaldism
2

Waardernburg syndrome
3

Cronkhite-Canada syndrome
4

Vogt-Koyanagi-Harada syndrome
5

Cornelia de Lange Syndrome
Q/Q(M)-474566 Report a Problem


The combination of poliosis, uveitis, deafness, and vitiligo are characteristic of which of the following
conditions?
4

Vogt-Koyanagi-Harada syndrome
Vogt-Koyanagi-Harada syndrome is characterized by poliosis, uveitis, deafness, and Vitiligo.
Q/Q(M)-474566 Report a Problem


Tricholemmoma is which type of hair follicle tumor?
1

Tumor of the hair germ cells
2

Tumors of matrical differentiation
3

Tumors of the external root sheath
4

Tumor of isthmus differentiation
5

Tumor of the internal root sheath
Q/Q(M)-482640 Report a Problem


Tricholemmoma is which type of hair follicle tumor?
3

Tumors of the external root sheath
Tricholemmoma is a tumor of the external root sheath. Tumors of the hair germ cells include
trichoblastoma and trichoepithelioma. Tumors of matrical differentiation include pilomatricoma. Tumors
of the external root sheath include pilar cyst, proliferating tricholemmal cyst, tricholemmoma,
tricholemmal carcinoma. Tumor of the isthmus differentiation include tumor of the follicular
infundibulum.
Q/Q(M)-482640 Report a Problem


A 42 year-old woman presents with the complaint of excess hair growth on her face. She has normal
menses and has recently had her "annual" exam and the note relates normal sized ovaries. What is the
most logical next step?
1

Check plasma levels of androstenedione and testosterone
2

Send a 21-hydroxylase enzyme deficiency test
3

Biopsy from the most affected area
6

4

Refer her to endocrinology
5

Order a CT of the abdomen
Q/Q(M)-478083 Report a Problem



A 42 year-old woman presents with the complaint of excess hair growth on her face. She has normal
menses and has recently had her "annual" exam and the note relates normal sized ovaries. What is the
most logical next step?
1

Check plasma levels of androstenedione and testosterone
Women with idiopathic hirsutism will have evidence of androgen excess but with normal menses, normal-
sized ovaries and no evidence of tumors of adrenal or ovary and normal adrenal function. They will often
have slight elevations of plasma androstenedione and testosterone. Check the blood levels of the plasma
steroids would be a logical first step.
Q/Q(M)-478083 Report a Problem


Spotted red lunulae are absent in which of the following conditions?
1

Alopecia areata
2

Keratosis follicularis
3

Systemic lupus erythematosus
4

Rheumatoid arthritis
5

Lichen planus
Q/Q(M)-478099 Report a Problem
Spotted red lunulae are absent in which of the following conditions?
2

Keratosis follicularis
Keratosis follicularis has nail findings of v-shaped nicking of the distal nail, subungual hyperkeratosis and
red/white alternating longitudinal bands. In addition to those listed above, psoriasis can also cause spotted
red lunulae.
Q/Q(M)-478099 Report a Problem

Proximal white subungual onychomycosis with polydactylous involvement needs at least one laboratory
test:
1

Blood count
2

Sedimentation rate
3

T-helper CD4 lymphocytes
4

Liver enzymes
5

HIV
Q/Q(M)-475895 Report a Problem

Proximal white subungual onychomycosis with polydactylous involvement needs at least one laboratory
test:
7

5

HIV
This type of onychomycosis has a prognostic value: It means that the CD4 lymphocytes are below 450.
Q/Q(M)-475895 Report a Problem


The syndrome including mental and physical retardation, convulsions, episodic unconsciousness, liver
enlargement, skin lesions, and dry and brittle hair showing trichorrhexis nodosa microscopically and
fluorescing red. is:
1

Bazex Follicular Atrophoderma
2

Crandall's syndrome
3

Citrullinemia
4

Argininosuccinic aciduria
5

Bjornstad's syndrome
Q/Q(M)-478058 Report a Problem


The syndrome including mental and physical retardation, convulsions, episodic unconsciousness, liver
enlargement, skin lesions, and dry and brittle hair showing trichorrhexis nodosa microscopically and
fluorescing red. is:
4

Argininosuccinic aciduria
Argininosuccinic aciduria has the features listed including trichorrhexis nodosa and red fluorescence of
the hair. Bazex's follicular atrophoderma and Crandall syndrome both have findings of pili torti and
citrullinemia has trichorrhexis nodosa. None of the other options have red fluorescence as a feature.
Q/Q(M)-478058 Report a Problem

Distal subungual onychomycosis is most often caused by:
1

Trichophyton mentagrophytes
2

Epidermophyton floccosum
3

Trichophyton schoenleinii
4

Trichophyton rubrum
5

Trichophyton megninii
Q/Q(M)-473973 Report a Problem


Distal subungual onychomycosis is most often caused by:
4

Trichophyton rubrum
Trichophyton rubrum is the most common organism involved in distal subungual onychomycosis.
Q/Q(M)-473973 Report a Problem



Mee's lines are:
1

Double white transverse lines from abnormal vascular bed
8

2

Brownish macules beneath the nail plate
3

Local or diffuse hyperkeratotic tissue that develops on the lateral or proximal nail folds
4

Transverse white lines that affect all nails, grow out with nail growth
5

Vertical black lines on a single or multiple nails
Q/Q(M)-478123 Report a Problem



Mee's lines are:
4

Transverse white lines that affect all nails, grow out with nail growth
Mee's lines are transverse white lines that affect all nails, grow out with nail growth. They are seen in
arsenic poisoning, rheumatic fever, congenital heart failure, leprosy and with significant systemic disease.
Brownish macules beneath the nail plate are oil spots, seen in psoriasis. Onychophosis is the local or
diffuse hyperkeratotic tissue that develops on the lateral or proximal nail folds. The double white
transverse lines from abnormal vascular bed are Muehrcke's lines which are caused by a nephrotic
syndrome, low albumin, liver disease or malnutrition.
Q/Q(M)-478123 Report a Problem


Frontal bossing, saddle nose, hypoplastic midface, peg shaped or conical teeth and hypopigmented short
sparse scalp and body hair are prominent features of which of the following
1

Christ Siemens Touraine syndrome
2

Hidrotic ectodermal dysplasia
3

Arginosuccinic aciduria
4

Monilethrix
5

Pachyonychia congenita
Q/Q(M)-477678 Report a Problem

Frontal bossing, saddle nose, hypoplastic midface, peg shaped or conical teeth and hypopigmented short
sparse scalp and body hair are prominent features of which of the following
1

Christ Siemens Touraine syndrome
Chrsit Siemens Touraine syndrome also known as Hypohidrotic ectodermal dysplaisa or anhidrotic
ectodermal dysplasia is X-linked recessive disroder that presents in infancy to early childhood. Features
include hypo-anhidrosis with increased body temperature, hypopigmented sparse scalp and body hair,
frontal bossing, saddle nose, hypoplastic midface, abnormal ears, hypo-anodontia, and increased
bronchopulmonary inections. Cloustons syndrome or hidrotic ectodermal dysplasia is AD and associated
with palmoplantar keratoderma, nail dystrophy, short sparse scalp hair, and tufting of the terminal
phalanges. Arginosuccinic aciduria is associated with seizures, trichorrhexis nodosa, failure to thrive, and
hyperammonemia. Monelothrix is associated with beaded hairs and keratosis pilaris. Pachyonychia
congenita is associated with nail dystrophy and focal palmoplantar keratoderma.
Q/Q(M)-477678 Report a Problem


A 73-year-old man presents with a red streak involving the nail plate of the first digit. You make the
diagnosis of longitudinal erythronychia. What is the most common underlying cause:
9

1

Melanoma
2

Myxoid cyst
3

Subungual keratoacanthoma
4

Carbon Monoxide toxicity
5

Onychopapilloma
Q/Q(M)-482924 Report a Problem


A 73-year-old man presents with a red streak involving the nail plate of the first digit. You make the
diagnosis of longitudinal erythronychia. What is the most common underlying cause:
5

Onychopapilloma
Onychopapilloma represents a benign tumor of the nail unit and is the most common etiology of
longitudinal melanonychia. Myxcoid cysts are mucin-filled pseudocysts that may be connected to the joint
space and often present as longitudinal grooving of the nail plate. Carbon Monoxide toxicity may present
with a red lunula.
Q/Q(M)-482924 Report a Problem
A 36-year-old woman presents with hair loss. Examination of a single follicle reveals multiple hair shafts
protruding, each surrounded by its own cuticle. What is the most likely diagnosis:
1

Pili multigemini
2

Pili bifurcati
3

Trichoptilosis
4

Trichoschisis
5

Trichothiodystrophy
Q/Q(M)-482942 Report a Problem

A 36-year-old woman presents with hair loss. Examination of a single follicle reveals multiple hair shafts
protruding, each surrounded by its own cuticle. What is the most likely diagnosis:
1

Pili multigemini
Pili multigemini refers to a rare disorder of multiple divided hair matrices, with multiple hair shafts each
with their own cuticle all arising from one follicle. Pili bifucarti exhibits hair shaft bifurcation into short
segments along hair shafts, with each segment covered with its own cuticle. Trichoptilosis refers to
split ends of the distal portion of the hair shaft. Trichoschisis is a clean fracture of the hair shaft.
Trichothiodystrophy is a disorder of beaded hair when viewed with polarizing light.
Q/Q(M)-482942 Report a Problem


Pili trianguli et canaliculi is the characteristic hair fiding in which of the following disoders?
1

Monilethrix
2

Pili annulati
3

Naxos syndrome
4

Anhidrotic ectodermal dysplasia
10

5

Uncombable hair syndrome
Q/Q(M)-474553 Report a Problem

Pili trianguli et canaliculi is the characteristic hair fiding in which of the following disoders?
5

Uncombable hair syndrome
Pili trianguli et canaliculi, characterized by a triangular hair shaft with a central longitudinal groove, is the
finding seen in uncombable hair syndrome.
Q/Q(M)-474553 Report a Problem


The combination of a low-set hairline and synophrys is seen in which of the following conditions?
1

Werner syndrome
2

Turner Syndrome
3

Noonan Syndrome
4

Cornelia de Lange Syndrome
5

Waardenburg's syndrome
Q/Q(M)-474563 Report a Problem


The combination of a low-set hairline and synophrys is seen in which of the following conditions?
4

Cornelia de Lange Syndrome
Cornelia de Lange Syndrome is a sporadic disorder that may be caused by mutations in nipped-Beta-like
gene. Clinically, patients have cutis marmorata, hirsutism, synophrys, mental retardation, short stature,
GU abnormalities, hearing loss, and congenital heart defects.
Q/Q(M)-474563 Report a Problem


What is the gene defect that causes red hair?
1

Tyrosinase
2

P protein
3

Melanocortin 1 receptor
4

Tyrosine aminotransferase
5

Tyrosinase-related protein 1
Q/Q(M)-476535 Report a Problem


What is the gene defect that causes red hair?
3

Melanocortin 1 receptor
Defects in melanocortin 1 receptor lead to the phenotypic appearance of red hair. Individuals with red hair
have increased ratio of pheomelanin to eumelanin and are at increased risk for melanoma.
Q/Q(M)-476535 Report a Problem


11


Yellow nail syndrome has been associated with use of which of the following treatments for Rheumatoid
Arthritis?
1

D-penicillamine
2

Methotrexate
3

Infliximab
4

Etanercept
5

Adalimumab
Q/Q(M)-478094 Report a Problem


Yellow nail syndrome has been associated with use of which of the following treatments for Rheumatoid
Arthritis?
1

D-penicillamine
D-penicillamine has been associated with the development of yellow nail syndrome in patiens with
rheumatoid arthritis. The other options are used to treat RA, but have not been associated with
development of this syndrome.
Q/Q(M)-478094 Report a Problem

Which of the following is caused by mutations in gap junction proteins?
1

Rothmund-Thompson syndrome
2

Hidrotic ectodermal dysplasia
3

Anhidrotic ectodermal dysplasia
4

Netherton syndrome
5

Naxos syndrome
Q/Q(M)-474555 Report a Problem


Which of the following is caused by mutations in gap junction proteins?
2

Hidrotic ectodermal dysplasia
Hidrotic ectodermal dysplasia is caused by mutations in connexin 30, which is a gap junction protein.
Q/Q(M)-474555 Report a Problem


Congenital generalized hypertrichosis is associated with which of the following symptoms?
1

Androgen-secreting ovarian tumors
2

Pituitary insufficiency
3

Distichiasis
4

Methimazole teratogenicity
5

Gingival fibromatosis
Q/Q(M)-474574 Report a Problem
12


Congenital generalized hypertrichosis is associated with which of the following symptoms?
5

Gingival fibromatosis
Gingival fibromatosis. Congenital Generalized Hypertrichosis with Gingival Fibromatosis is a genetic
condition with hypertrichosis on the face and upper body and gingival hyperplasia and fibromatosis.
Q/Q(M)-474574 Report a Problem


Which of the following is an example of an apparent leukonychia?
1

Terry's lines
2

Mee's lines
3

Muehrcke's lines
4

Hutchinson's lines
5

Onychoschizia
Q/Q(M)-476751 Report a Problem


Which of the following is an example of an apparent leukonychia?
3

Muehrcke's lines
An apparent leukonychia disappears with pressure and is a problem with nail bed, not nail plate. Apparent
leukonychia may be indicative of overall health compromise. Muehrcke's lines are paired white bands and
are a type of apparent leukonychia.
Q/Q(M)-476751 Report a Problem


Trichobezoar is is a potentially life-threatening complication of:
1

Alopecia areata
2

Anhidrotic ectodermal dysplasia
3

Trichotillomania
4

Discoid lupus
5

Lichen planopilaris
Q/Q(M)-477345 Report a Problem



Trichobezoar is is a potentially life-threatening complication of:
3

Trichotillomania
Trichophagia is more common in individuals who have richotillomania. This chewing behavior frequently
can lead to the formation of trichobezoars in the stomach or small intestines. Trichobezoars can result in
anemia, abdominal pain, hematemesis, nausea and/or vomiting, bowel obstruction, perforation,
gastrointestinal (GI) bleeding, acute pancreatitis, and obstructive jaundice.
Q/Q(M)-477345 Report a Problem

The "ragged cuticle" seen in dermatomyositis is also known as:
13

1

Samitz sign
2

Ventral pterygium
3

Plummers sign
4

Candy-cane nails
5

Emperipolesis
Q/Q(M)-478101 Report a Problem



The "ragged cuticle" seen in dermatomyositis is also known as:
1

Samitz sign
Cuticular fraying from dermatomyositis is called the Samitz sign or "ragged cuticle". The other options are
not associated with dermatomyositis. A ventral pterygium can be seen in systemic sclerosis. There is no
such sign as Plummers sign. Candy-cane nails are seen in Darier's disease. Emperipolesis is seen in Rosai-
Dorfman disease and is not a nail finding.
Q/Q(M)-478101 Report a Problem


This nail finding is characteristically associated with what disorder?
1

Congenital heart failure
2

Chronic renal failure
3

Anemia
4

Hyperthyroidism
5

Cirrhosis
Q/Q(M)-475940 Report a Problem

This nail finding is characteristically associated with what disorder?
2

Chronic renal failure
Lindsays nail is characteristically associated with chronic renal failure.
Q/Q(M)-475940 Report a Problem


The defect associated with the hair finding shown in the image is:
14

1

hHB6/hHB1
2

hHB7/hHB2
3

hHB8/hHB3
4

hHB9/hHB4
5

hHB10/hHB5
Q/Q(M)-478073 Report a Problem



The defect associated with the hair finding shown in the image is:
1

hHB6/hHB1
hHB6/hHB1 keratins are associated with Monilethrix. Other findings include: short, brittle hair, extensor
keratosis pilaris, mental retardation, syndactyly, cataracts, and nail/teeth abnormalities.
Q/Q(M)-478073 Report a Problem


Which autoantibody is associated with these cuticular changes, shawl sign, and overall favorable
prognosis?
1

Mi-2
2

Jo-1
3

High titered ANA with speckled pattern
4

Scl-70
5

Ro
Q/Q(M)-476588 Report a Problem


Which autoantibody is associated with these cuticular changes, shawl sign, and overall favorable
prognosis?
1

Mi-2
Dermatomyositis has many different cutaneous manifestations including heliotrope rash, Gottron's papules
and sign, mechanic's hand and poikiloderma atrophicans vasculare. Antibodies to Mi-2 in dermatomyositis
are associated with cuticular changes, shawl sign and a overall favorable prognosis.
Q/Q(M)-476588 Report a Problem

A 26-year-old woman presents with hair loss. Examination of a single shaft reveals division into two
shafts in short segments, with each individually covered with a complete cuticle. What is the most likely
diagnosis:
15

1

Pili multigemini
2

Pili bifurcati
3

Trichoptilosis
4

Trichoschisis
5

Trichothiodystrophy
Q/Q(M)-482941 Report a Problem



A 26-year-old woman presents with hair loss. Examination of a single shaft reveals division into two
shafts in short segments, with each individually covered with a complete cuticle. What is the most likely
diagnosis:
2

Pili bifurcati
Pili multigemini refers to a rare disorder of multiple divided hair matrices, with multiple hair shafts each
with their own cuticle all arising from one follicle. Pili bifucarti exhibits hair shaft bifurcation into short
segments along hair shafts, with each segment covered with its own cuticle. Trichoptilosis refers to
split ends of the distal portion of the hair shaft. Trichoschisis is a clean fracture of the hair shaft.
Trichothiodystrophy is a disorder of beaded hair when viewed with polarizing light.
Q/Q(M)-482941 Report a Problem


Endonyx onychomycosis is due to:
1

Trichophyton rubrum
2

Trichophyton mentagrophytes
3

Trichophyton soudanense
4

Scytalidium dimidiatum
5

Scopuliaropsis
Q/Q(M)-475893 Report a Problem



Endonyx onychomycosis is due to:
3

Trichophyton soudanense
Endonyx onychomycosis may also be observed with T. violaceum. This is the equivalent of endothrix
infection of the hair associated with tinea capitis.
Q/Q(M)-475893 Report a Problem


A patient with psoriasis has pitting of the nails. This finding is due to involvement of which part of the
nail unit?
1

Proximal matrix
2

Nail bed
3

Hyponychium
4

Proximal nail fold
16

5

Distal matrix
Q/Q(M)-477350 Report a Problem


A patient with psoriasis has pitting of the nails. This finding is due to involvement of which part of the
nail unit?
1

Proximal matrix
Disease in the proximal matrix is responsible for producing pitting, onychorrhexis, and Beaus lines.
Changes in the intermediate matrix can cause leukonychia, and the distal matrix may be responsible for
focal onycholysis, thinned nail plate, and erythema of the lunula.
Q/Q(M)-477350 Report a Problem


Muehrcke's nails are most often associated with:
1

Hyperthyroidism
2

Hypothyroidism
3

Hypoalbuminemia
4

Hyperalbuminemia
5

None of these answers are correct
Q/Q(M)-478687 Report a Problem


Muehrcke's nails are most often associated with:
3

Hypoalbuminemia
Muehrcke's nails are most associated with hypoalbuminemia. They represent transverse double white lines
that are the abnormality of the vascular bed, probably a localized edematous state secondary to the
hypoalbuminemia.
Q/Q(M)-478687 Report a Problem


A patient with wooly hair, diffuse palmoplantar keratoderma, and right ventricular arrhythmogenic
cardiomyopathy likely has a mutation in which of the following genes?
1

HHB6
2

Plakoglobin
3

Plakophilin
4

SPINK 5
5

SLURP 1
Q/Q(M)-474554 Report a Problem


A patient with wooly hair, diffuse palmoplantar keratoderma, and right ventricular arrhythmogenic
cardiomyopathy likely has a mutation in which of the following genes?
2

Plakoglobin
Naxos syndrome is caused by mutations in the plakoglobin gene. Its clinical features include wooly hair,
17

diffuse palmoplantar keratoderma, and right ventricular arrhythmogenic cardiomyopathy.
Q/Q(M)-474554 Report a Problem



Anonychia is not a feature of which of the following syndromes?
1

Nail-Patella syndrome
2

Coffin-Siris syndrome
3

DOOR syndrome
4

COIF syndrome
5

Hidrotic ectodermal dysplasia
Q/Q(M)-478127 Report a Problem


Anonychia is not a feature of which of the following syndromes?
5

Hidrotic ectodermal dysplasia
Hidrotic ectodermal dysplasia features hyperconvex nails, micronychia and nail dystrophy. All of the
other listed options can cause anonychia. COIF syndrome is Congenital Onychodysplasia of the Index
Finger and DOOR syndrome is Deafness, Congenital Onychodystrophy Recessive Form. Coffin-Siris
syndrome is characterized by: nail onychodysplasia or aplasia (usually of the fifth finger or toe), coarse
facies including bushy eyebrows, scant scalp hair, full lips, and microcephaly, mental/growth deficiency,
short distal phalanges, and other abnormalities.
Q/Q(M)-478127 Report a Problem


Yellow nail syndrome can be associated with:
1

D-penicillamine use in patients with rheumatoid arthritis
2

Wilsons disease
3

Alopecia areata
4

Doxorubicin
5

Minocycline use in acne patients
Q/Q(M)-473972 Report a Problem


Yellow nail syndrome can be associated with:
1

D-penicillamine use in patients with rheumatoid arthritis
D-penicillamine use in patients with rheumatoid arthritis. Yellow nail syndrome has been described in
patients with rheumatoid arthritis receiving thiol compounds including D-penicillamine and bucillamine.
Reversal of the condition has been described after discontinuation of the drug.
Q/Q(M)-473972 Report a Problem

Best diagnosis of this type of cicatricial alopecia shown in attached image is
1

Chronic cutaneous lupus erythematosus
18

2

Lichen planopilaris
3

Central centrifugal cicatricial alopecia
4

Pseudopelade of Brocq
5

Alopecia mucinosa
Q/Q(M)-482063 Report a Problem


Best diagnosis of this type of cicatricial alopecia shown in attached image is
3

Central centrifugal cicatricial alopecia
Central centrifugal cicatricial alopecia is a new term adopted by the NAHRS consensus group to
encompass the terms hot comb alopecia, follicular degeneration syndrome, pseudopelade in African
Americans, and central elliptical pseudopelade in Caucasians.In general, it is a condition that presents with
flesh-colored, noninflammatory cicatricial alopecia of the central scalp that, over time, enlarges
centrifugally. Pseudopelade of Brocq can mimic alopecia areata, lichen planopilaris, and DLE. In addition,
central centrifugal cicatricial alopecia (CCCA), a noninflammatory condition of the central scalp that is
seen primarily in black women. CCCA usually presents as a symmetric patch, unlike the irregularly
bordered, typically atrophic plaques seen in classic pseudopelade of Brocq
Q/Q(M)-482063 Report a Problem



A 64-year-old man admitted for an exacerbation of his severe congestive heart failure exhibits blanchable
white horizontal lines beneath the nail plates of all digits. What is the most likely disorder:
1

Beaus lines
2

Mees lines
3

Muehrckes lines
4

Onychophagia
5

Dolichonychia
Q/Q(M)-482947 Report a Problem


A 64-year-old man admitted for an exacerbation of his severe congestive heart failure exhibits blanchable
white horizontal lines beneath the nail plates of all digits. What is the most likely disorder:
3

Muehrckes lines
Beaus lines are transverse indented nail plate furrows caused by temporary growth arrest of the nail
matrix, often due to chemotherapy or other stressful events and illnesses. Mees lines are transverse
white lines affecting all nails and growing out with the nail; there is no associated indentation. Mees
19

lines may be caused by arsenic poisoning, rheumatic fever, or other systemic diseases. Muehrckes lines
disappear with squeezing of the nail and are due to disorders of low albumin. Oncyhophagia refers to nail
biting. Dolichonychia exhibits long, slender nails, and is associated with Ehlers-Danlos and Marfans.
Q/Q(M)-482947 Report a Problem



A rapid onset of hair growth with or without accompanying virilization can occur in all of the following
conditions EXCEPT:
1

Adrenal adenoma
2

Adrenal carcinoma
3

Arrhenoblastoma
4

Krukenburg tumor of the ovary
5

Ovarian cyst
Q/Q(M)-478078 Report a Problem


A rapid onset of hair growth with or without accompanying virilization can occur in all of the following
conditions EXCEPT:
5

Ovarian cyst
All of these options except an ovarian cyst can cause a rapid onset of hair growth with or without
accompanying virilization. An Arrhenoblastoma is a tumor of the ovary that secretes testosterone.
Q/Q(M)-478078 Report a Problem


Acquired progressive kinking of hair is a complication of which of the following medications?
1

Antimetabolites
2

Antimalarials
3

Retinoids
4

Slfonamides
5

Dapsone
Q/Q(M)-474569 Report a Problem

Acquired progressive kinking of hair is a complication of which of the following medications?
3

Retinoids
Retinoids can cause acquired progressive kinking of hair.
Q/Q(M)-474569 Report a Problem

The epithelium that lies on the volar surface of the digit is the:
1

Nail bed
2

Nail matrix
20

3

Nail plate
4

Hyponychium
5

Eponychium
Q/Q(M)-478090 Report a Problem


The epithelium that lies on the volar surface of the digit is the:
4

Hyponychium
The epithelium that lies on the volar surface of the digit is the hyponychium. The nail plate is the actual
nail made up of tightly packed onychocytes. The nail matrix is the epithelium which starts mid distal
phalanx which generated the nail plate. It keratinizes without a granular layer and determines the thickness
of the nail plate. The nail bed is the thin epithelium immeduately beneath the nail plate.
Q/Q(M)-478090 Report a Problem

A subungual hematoma covering 50% of the nailbed should be managed by which of the following?
1

No treatment
2

Removal of the nail plate
3

Trephination of the nail plate
4

Needle aspiration of the hematoma
5

Pressure dressing
Q/Q(M)-474587 Report a Problem


A subungual hematoma covering 50% of the nailbed should be managed by which of the following?
2

Removal of the nail plate
Removal of the nail plate is necessary for management of hematomas covering more than 25% of the
nailbed.
Q/Q(M)-474587 Report a Problem


The most common enzyme abnormality in congenital adrenal hyperplasia is:
1

3-beta-hydroxysteroid dehydrogenase isomerase
2

11-beta-hydroxylase
3

21-hydroxylase
4

15-hydroxylase
5

17-hydroxylase
Q/Q(M)-478082 Report a Problem


The most common enzyme abnormality in congenital adrenal hyperplasia is:
3

21-hydroxylase
21-hydroxylase deficiency is present in 95% of cases of congenital adrenal hyperplasia. This defect in
21

adrenal steroidogenesis can occur at any point in life, but affected girls will generally present around
puberty with hirsutism and mentsrual irregularity/primary amenorrhea. The key feature is excess androgen
production.
Q/Q(M)-478082 Report a Problem


Mutations in c-kit are associated with which of the following conditions?
1

Waardenburg syndrome
2

Piebaldism
3

Tuberous sclerosis
4

Vogt-koyanagi-harada
5

Cornelia de Lange Syndrome
Q/Q(M)-474565 Report a Problem

Mutations in c-kit are associated with which of the following conditions?
2

Piebaldism
Piebaldism is caused by mutations in c-kit.
Q/Q(M)-474565 Report a Problem


A patient with cirrhosis has noticed that the proximal aspect of his nails have become white. This
characteristic finding is called:
1

Half-and-half nails
2

Mees' lines
3

Beau's lines
4

Terry's lines
5

Muehrcke's lines
Q/Q(M)-477379 Report a Problem

A patient with cirrhosis has noticed that the proximal aspect of his nails have become white. This
characteristic finding is called:
4

Terry's lines
Terrys lines are described as an abnormal white appearance of the nail except for the most distal
portion. It has been described in association with cirrhosis, congestive heart failure, and diabetes mellitus.
Q/Q(M)-477379 Report a Problem


A woman with hypopigmented in lines of Blaschko and scarring alopecia likely suffers which of the
following conditions?
1

Chondrodysplasis punctata
2

Anhidrotic Ectodermal Dysplasia
22

3

Focal Dermal Hypoplasia
4

Rothmund-Thomson Syndrome
5

Bloch-Sulzberger Syndrome
Q/Q(M)-474560 Report a Problem


A woman with hypopigmented in lines of Blaschko and scarring alopecia likely suffers which of the
following conditions?
5

Bloch-Sulzberger Syndrome
Bloch-Sulzberger Syndrome (incontinentia pigmenti).
Q/Q(M)-474560 Report a Problem


This characteristic nail finding is seen in what disorder?
1

Lichen planus
2

Pachyonychia congenita
3

Darier's disease
4

COIF syndrome
5

Coffin-Siris syndrome
Q/Q(M)-475937 Report a Problem


This characteristic nail finding is seen in what disorder?
3

Darier's disease
Alternating red and white longitudinal bands on the nail are characteristic of Dariers disease. Other nail
findings include subungual hyperkeratosis and distal v-shaped nicking.
Q/Q(M)-475937 Report a Problem

What nutritional deficiency can lead to this nail finding?
23

1

Vitamin B12
2

Vitamin C
3

Iron
4

Magnesium
5

Thiamine
Q/Q(M)-475939 Report a Problem


What nutritional deficiency can lead to this nail finding?
3

Iron
Iron deficiency can lead to spoon nail deformity or koilonychias.
Q/Q(M)-475939 Report a Problem



What is the common cause of the one hand-two-foot syndrome?
1

Trichophyton rubrum
2

Trichophyton mentagrophytes ( var. Interdigitale)
3

Candida albicans
4

Scytalidium dimidiatum
5

Scopulariopsis brevicaulis
Q/Q(M)-475890 Report a Problem



What is the common cause of the one hand-two-foot syndrome?
1

Trichophyton rubrum
This condition is due to T. rubrum in 90% of the cases , T. interdigitale and Scytalidium dimidiatum share
the 10% left.
Q/Q(M)-475890 Report a Problem


24

The most common cultured agent in chronic paronychia associated with a black hue of the lateral edge of
the nail plate is:
1

Trichophyton rubrum
2

gram-negative bacteria
3

Candida spp
4

Scytalidium dimidiatum
5

Scopulariopsis brevicaulis
Q/Q(M)-475897 Report a Problem

The most common cultured agent in chronic paronychia associated with a black hue of the lateral edge of
the nail plate is:
3

Candida spp
Chronic paronychia is likely a condition that represents a contact reaction to allergens or irritants. It often
is superinfected with candida. This underlies the importance of treatment with both an antifungal and a
topical steroid.
Q/Q(M)-475897 Report a Problem


Which of the following is not a feature of Bazex-Dupre-Christol Syndrome?
1

peripheral vasodilation with cyanosis
2

BCC
3

hypohidrosis
4

milia
5

epidermoid cysts
Q/Q(M)-482918 Report a Problem


Which of the following is not a feature of Bazex-Dupre-Christol Syndrome?
1

peripheral vasodilation with cyanosis
Peripheral vasodilation with cyanosis is not found in BDC Syndrome. This is a feature of Rombo
Syndrom. Bazex-Dupre-Christol Syndrome includes BCC, follicular atrophoderma, hypohidrosis, milia,
epidermoid cysts, and hypotrichosis.
Q/Q(M)-482918 Report a Problem


A commonly known cause of splinter hemorrhages in the nail is endocarditis. Which of the following
would be unlikely to cause splinter hemorrhages?
1

Trichinosis
2

Trauma
3

Psoriasis
4

Vasculitis
5

Rheumatoid arthritis
25

Q/Q(M)-478100 Report a Problem


A commonly known cause of splinter hemorrhages in the nail is endocarditis. Which of the following
would be unlikely to cause splinter hemorrhages?
5

Rheumatoid arthritis
Rheumatoid arthritis is not a cause of splinter hemorrhages of the nail. Endocarditis is the most commonly
thought of cause of splinter hemorrhages. Trichinosis, trauma, psoriasis and vasculitis also can be causes
of this also and should be considered when splinter hemorrhages are seen.
Q/Q(M)-478100 Report a Problem
Which of the following medication is a cause of hirsutism without virilization?
1

Dapsone
2

Diazoxide
3

Diazepam
4

Dinitrochlorobenzene
5

Dantrolene
Q/Q(M)-478079 Report a Problem

Which of the following medication is a cause of hirsutism without virilization?
2

Diazoxide
Phenytoin, diazoxide, cyclosporine and hexachlorobenzene all can cause increased hair growth in patients.
In addition, oral (and topical) minoxidil can cause hirsutism. The other listed medications do not cause
hirsutism.
Q/Q(M)-478079 Report a Problem


The distal nail matrix is visible at the following portion of the nail unit:
1

Onychodermal band
2

Hyponychium
3

Lunula
4

Cuticle
5

Lateral nailfold
Q/Q(M)-482570 Report a Problem


The distal nail matrix is visible at the following portion of the nail unit:
3

Lunula
The distal nail matrix is located at the lunula, which is visible as the half-moon structure at the base of the
nail. The proximal matrix is located under the proximal nailfold. Damage to the proximal nail matrix
produces defects at the dorsal nailplate, and damage to the distal nail matrix produces produces defects at
the ventral nail plate.
Q/Q(M)-482570 Report a Problem
26




The visible portion of the nail matrix is called the:
1

Eponychium
2

Lunula
3

Cuticle
4

Hyponychium
5

Nail plate
Q/Q(M)-477229 Report a Problem


The visible portion of the nail matrix is called the:
2

Lunula
The nail unit is comprised of six major components including the nail matrix, nail plate, cuticle, nail bed,
anchoring portion, and the framing portion (proximal/lateral/distal folds). The lunula is a component of
the distal matrix. It is grayish white because of the nature of the keratinization of its epithelium.
Q/Q(M)-477229 Report a Problem


A 6-year old girl with atopic dermatitis presents with patchy non-scarring alopecia and nail pitting. You
make the diagnosis of alopecia areata. What portion of the nail unit is responsible for her nail plate pitting:
1

Hyponychium
2

Proximal nail plate
3

Proximal nail matrix
4

Distal nail matrix
5

Eponychium
Q/Q(M)-482930 Report a Problem



A 6-year old girl with atopic dermatitis presents with patchy non-scarring alopecia and nail pitting. You
make the diagnosis of alopecia areata. What portion of the nail unit is responsible for her nail plate pitting:
3

Proximal nail matrix
The proximal nail matrix produce the dorsal nail plate. Defective keratinization in this region is therefore
responsible for the pitting seen in alopecia areata, psoriasis, eczema, and other cutaneous diseases. The
distal nail matrix produces the ventral nail plate and is visible as the lunula.
Q/Q(M)-482930 Report a Problem

Acute paronychia is most commonly caused by which of the following organisms?
1

Candida albicans
2

Candida tropicalis
27

3

Pseudomonas aeruginosa
4

Staphylococcus aureus
5

Trichophyton rubrum
Q/Q(M)-474580 Report a Problem

Acute paronychia is most commonly caused by which of the following organisms?
4

Staphylococcus aureus
Staphylococcus aureus is the most common cause of acute paronychia.
Q/Q(M)-474580 Report a Problem


Thin hair with premature graying is characteristic of which of the following syndromes?
1

Werner syndrome
2

Turner Syndrome
3

Noonan Syndrome
4

Cornelia de Lange Syndrome
5

Waardenburg's syndrome
Q/Q(M)-474562 Report a Problem


Thin hair with premature graying is characteristic of which of the following syndromes?
1

Werner syndrome
Werner syndrome is characterized by premature degeneration of major organs that loosely mimics
premature aging. It is associated with sclerodermoid changes, leg ulcerations, premature graying,
cataracts, premature atherosclerosis,a dn diabetes. It is associated with defects in the helicase RecQL2.
Q/Q(M)-474562 Report a Problem


A 32-year-old male presents with progressive curliness of the temporal scalp hairs that seem to precede
hair loss. What is the most likely associated etiology?
1

Secondary syphilis
2

Systemic tuberculosis
3

Lichen planopilaris
4

Hodgkins lymphoma
5

HIV
Q/Q(M)-482935 Report a Problem

A 32-year-old male presents with progressive curliness of the temporal scalp hairs that seem to precede
hair loss. What is the most likely associated etiology?
5

HIV
HIV and retinoids are associated with acquired progressive kinking. This syndrome presents with gradual
28

curling and darkening of hair in the frontal and temporal regions of the scalp, followed by androgenetic
alopecia. Secondary syphilis may present as a non-scarring alopecia, typically in a diffuse pattern.
Q/Q(M)-482935 Report a Problem


Ferritin levels need to be, at minimum, above which of the serum levels to treat iron-deficiency related
telogen effluvium?
1

20ng/dL
2

40ng/dL
3

60ng/dL
4

30ng/dL
5

10ng/dL
Q/Q(M)-478206 Report a Problem


Ferritin levels need to be, at minimum, above which of the serum levels to treat iron-deficiency related
telogen effluvium?
2

40ng/dL
The therapeutic target for iron deficiency related telogen effluvium is greater than 40mg/dL. Other causes
of telogen effluvium include thyroid dysfunction, as well as such medications as beta-blockers,
antihyperlipidemic drugs, and NSAIDs.
Q/Q(M)-478206 Report a Problem

A patient with yellow nail syndrome should be evaluated for systemic disease involving which organ
system?
1

Hepatic
2

Renal
3

Pulmonary
4

Cardiovascular
5

Hematologic
Q/Q(M)-477364 Report a Problem


A patient with yellow nail syndrome should be evaluated for systemic disease involving which organ
system?
3

Pulmonary
Yellow nail syndrome is characterized by slowly growing, yellow nails that are excessively curved. The
lunulae and cuticles are absent. It has been associated with a array of pulmonary diseases including
tuberculosis, asthma, pleural effusion, bronchiectasis, sinusitis, bronchitis, and COPD.
Q/Q(M)-477364 Report a Problem

Black nail is caused by:
1

Trichophyton mentagrophytes
29

2

Proteus mirabilis
3

Pseudomonas spp.
4

Staphylococcus aureus
5

Hortaea werneckii
Q/Q(M)-478128 Report a Problem


Black nail is caused by:
2

Proteus mirabilis
Nail infections with Proteus mirabilis. T. mentag causes white discoloration of the nail and Pseudomonas
will cause a green or yellow/green discoloration. Hortaea werneckii causes tinea nigra. S. aureus does not
cause black nails.
Q/Q(M)-478128 Report a Problem
Primary total dystrophic onychomycosis is due to:
1

Candida spp
2

Fusarium
3

Trichophyton rubrum
4

Scytalidium dimidiatum
5

Scopulariopsis brevicaulis
Q/Q(M)-475898 Report a Problem


Primary total dystrophic onychomycosis is due to:
1

Candida spp
The main characteristic of this immunologic disorder the simultaneous involvement of all the tissues of
the nail unit.
Q/Q(M)-475898 Report a Problem


The combination of ankyloblepharon, ectodermal dysplasia, and cleft palate with wiry sparse hair is
characteristic of which of the following syndromes?
1

Focal Dermal Hypoplasia (Goltz syndrome)
2

Tricho-rhino-phalangeal Syndrome
3

Hay-Wells Syndrome
4

Ectrodactyly-Ectodermal dysplasia-Clefting (EEC)
5

Trichodentoosseous Syndrome
Q/Q(M)-474558 Report a Problem


The combination of ankyloblepharon, ectodermal dysplasia, and cleft palate with wiry sparse hair is
characteristic of which of the following syndromes?
3

Hay-Wells Syndrome
30

Hay-Wells Syndrome, also known as AEC (Ankyloblepharon-Ectodermal dysplasia-Clefting) is
characterized by wiry, sparse hair or alopecia, ankyloblepharon, PPK, partial anhidrosis, cleft lip, palate,
absent, and dystrophic nails.
Q/Q(M)-474558 Report a Problem


The most common cause for superficial type onychomycosis is:
1

Trichophyton rubrum
2

Trichophyton mentagrophytes
3

Epidermophyton floccosum
4

Fusarium oxysporum
5

Scopulariopsis brevicalis
Q/Q(M)-478093 Report a Problem


The most common cause for superficial type onychomycosis is:
2

Trichophyton mentagrophytes
T. mentagrophytes is the most common cause of superficial type onychomycosis. Other causes include:
Fusarium, Acremonium and Aspergillus spp.
Q/Q(M)-478093 Report a Problem



Which of the following are characteristics of Trichorhinophalangeal syndrome?
1

Kinky hair, frontal bossing, small widely spaced teeth with poor enamel
2

Palmoplantar keratoderma, trichorrhexis nodosa, sinus infections
3

Shortened phalanges, sparse hair, bulbous nose
4

Brittle hair, short stature, cerebellar ataxia
5

Sparse fine hair, Short extremities, Immunodeficiency
Q/Q(M)-477404 Report a Problem



Which of the following are characteristics of Trichorhinophalangeal syndrome?
3

Shortened phalanges, sparse hair, bulbous nose
Characteristic features of trichorhinophalangeal syndrome are shortened phalanges, sparse hair, and
bulbous nose.
Q/Q(M)-477404 Report a Problem

A 28-year-old male presents with three coin-shaped areas of scalp alopecia without scaling or erythema.
Which of the following is most likely to be associated with alopecia areata?
1

Hypothyroidism
31

2

Seborrheic dermatitis
3

Tinea capitis
4

Congestive heart failure
5

Chronic kidney disease
Q/Q(M)-482801 Report a Problem


A 28-year-old male presents with three coin-shaped areas of scalp alopecia without scaling or erythema.
Which of the following is most likely to be associated with alopecia areata?
1

Hypothyroidism
Alopecia area is associated with autoimmune diseases, especially thyroid disease, hypo- or
hyperthyroidism. It presents with well-defined patches of alopecia without scale or erythema. Seborrheic
dermatitis may result in temporary hair thinning, but does not result in coin-shaped patches of alopecia.
Tinea capitis presents with itchy scale on the scalp and possibly temporary poorly defined alopecia.
Congestive heart failure and chronic kidney disease are not associated with alopecia areata.
Q/Q(M)-482801 Report a Problem


Follicular atrophoderma is a feature of which of the following conditions?
1

Bazex syndrome
2

Menkes kinky hair syndrome
3

Papular atrichia
4

Tricho-dento-osseous syndrome
5

Tricho-rhino-phalangeal syndrome
Q/Q(M)-474552 Report a Problem


Follicular atrophoderma is a feature of which of the following conditions?
1

Bazex syndrome
Bazex syndrome is characterized by follicular atrophoderma, hypotrichosis, hypohidrosis, and mutiple
basal cell carcinomas of face.
Q/Q(M)-474552 Report a Problem



A patient has hyperkeratotic tissue on the lateral and proximal nails folds. What is the diagnosis?
1

Onychophosis
2

Onychomadesis
3

Hapalonychia
4

Onychocryptosis
5

Onychophagia
Q/Q(M)-482369 Report a Problem
32



A patient has hyperkeratotic tissue on the lateral and proximal nails folds. What is the diagnosis?
1

Onychophosis
Onychophosis describes hyperkeratosis tissue on the lateral and proximal nails folds. Onychomadesis is
shedding of the nail starting at the proximal end. Hapalonychia refers to soft, bendable nails.
Onychocryptosis refers to an ingrown toenails. Onychophagia means nail biting.
Q/Q(M)-482369 Report a Problem


A 44-year-old man undergoing chemotherapy for Hodgkins lymphoma presents with horizontal rough
bands involving the nail plates of all digits. What is the most likely disorder:
1

Beaus lines
2

Mees lines
3

Muehrckes lines
4

Onychophagia
5

Dolichonychia
Q/Q(M)-482945 Report a Problem



A 44-year-old man undergoing chemotherapy for Hodgkins lymphoma presents with horizontal rough
bands involving the nail plates of all digits. What is the most likely disorder:
1

Beaus lines
Beaus lines are transverse indented nail plate furrows caused by temporary growth arrest of the nail
matrix, often due to chemotherapy or other stressful events and illnesses. Mees lines are transverse
white lines affecting all nails and growing out with the nail; there is no associated indentation. Mees
lines may be caused by arsenic poisoning, rheumatic fever, or other systemic diseases. Muehrckes lines
disappear with squeezing of the nail and are due to disorders of low albumin. Oncyhophagia refers to nail
biting. Dolichonychia exhibits long, slender nails, and is associated with Ehlers-Danlos and Marfans.
Q/Q(M)-482945 Report a Problem


Which part of the nail complex results in nail pits?
1

Proximal nail fold
2

Proximal nail matrix
3

Dorsal nail matrix
4

Nail plate
5

Nail bed
Q/Q(M)-476750 Report a Problem



Which part of the nail complex results in nail pits?
2

Proximal nail matrix
33

Pitting of the nails are punctate depressions that migrate distally and result from defect in the proximal
nail matrix. It may be seen in conditions such as psoriasis, eczema and alopecia areata.
Q/Q(M)-476750 Report a Problem


A 24 year-old woman presents with the complaint of hirsutism and asks about therapy. She has a
virilization pattern of hair growth. On questioning, she relates having irregular menstrual periods and
acne. Which of the following is the most likely diagnosis?
1

Pheochromocytoma
2

Polycystic ovarian disease
3

Congenital adrenal hyperplasia
4

Ovarian carcinoma
5

Normal variant
Q/Q(M)-478081 Report a Problem


A 24 year-old woman presents with the complaint of hirsutism and asks about therapy. She has a
virilization pattern of hair growth. On questioning, she relates having irregular menstrual periods and
acne. Which of the following is the most likely diagnosis?
2

Polycystic ovarian disease
This patients diagnosis is most reasonably Polycystic Ovarian disease and includes the features listed
above in the question in addition to enlarged cystic ovaries, obesity and amenorrhea. Congenital adrenal
hyperplasia can present with hirsutism and menstrual abnormalities, but usually not obesity. Ovarian
carcinoma, pheochromocytoma and normal variant are not likely causes of this presentation.
Q/Q(M)-478081 Report a Problem


A patient with sparse hair, a pear-shaped broad nose and cone-shaped epiphyses likely suffers which of
the following conditions?
1

Focal Dermal Hypoplasia (Goltz syndrome)
2

Tricho-rhino-phalangeal Syndrome
3

Hay-Wells Syndrome
4

Ectrodactyly-Ectodermal dysplasia-Clefting (EEC)
5

Trichodentoosseous Syndrome
Q/Q(M)-474557 Report a Problem


A patient with sparse hair, a pear-shaped broad nose and cone-shaped epiphyses likely suffers which of
the following conditions?
2

Tricho-rhino-phalangeal Syndrome
Tricho-rhino-phalangeal Syndrome is characterized by sparse hair, a pear-shaped broad nose and cone-
shaped epiphyses.
Q/Q(M)-474557 Report a Problem


34


What determines the thickness of the nail plate?
1

Hyponychium
2

Nail matrix
3

Nail bed
4

Proximal nail fold
5

Nail plate
Q/Q(M)-475935 Report a Problem


What determines the thickness of the nail plate?
2

Nail matrix
The nail matrix determines the thickness of the nail plate.
Q/Q(M)-475935 Report a Problem


Which is the most effective for the treatment of onychomycosis?
1

Oral terbinafine
2

Terbinafine cream
3

Econazole cream
4

Clobetasol ointment
5

Oral ketoconazole
Q/Q(M)-482816 Report a Problem

Which is the most effective for the treatment of onychomycosis?
1

Oral terbinafine
Terbinafine, a squalene epoxidase inhibitor, is the treatment of choice for onychomycosis. The treatment
is with 250 mg by mouth daily for 12 weeks for toenails and 6 weeks for fingernails. Other treatment
options include oral itraconazole. Topicals such as terbinafine cream and econazole cream are good
choices for tinea pedis, but are not successful in treating onychomycosis. Topical corticosteroids such as
clobetasol ointment are not effective in treating fungal infections. Oral ketoconazole is ineffective for
onychomycosis.
Q/Q(M)-482816 Report a Problem


A significantly increased number of "club hairs" on a pull test is indicative of:
1

Anagen effluvium
2

Telogen effluvium
3

Normal hair anatomy
4

Alopecia areata
5

Angrogenetic alopecia
Q/Q(M)-478054 Report a Problem
35



A significantly increased number of "club hairs" on a pull test is indicative of:
2

Telogen effluvium
Telogen hairs are characteristically club shaped. An increased number of telogen hairs on hair pull would
be suggestive of telogen effluvium. Anagen hairs have a curled appearance at the root. While a small
number of telogen hairs are normal, a significantly increased number would not be normal hair anatomy.
Q/Q(M)-478054 Report a Problem


Terry's nails are seen in which of the following conditions?
1

Renal failure
2

Pulmonary fibrosis
3

Diabetes
4

Bladder carcinoma
5

Aerodigestive carcinoma
Q/Q(M)-478098 Report a Problem


Terry's nails are seen in which of the following conditions?
3

Diabetes
Terry's nails are described as all but the distal 2mm of each nail evenly white due to a defect in the nail
bed. It can be seen in cirrhosis, congenital heart failure and diabetes.
Q/Q(M)-478098 Report a Problem



Triangular lunulae are:
1

Part of the syndrome caused by a LMX1b mutation
2

Associated with hyperplastic patellae
3

Associated with a renal cell carcinoma
4

Associated with a mutation of ATP2C1
5

Associated with a mutation of SERCA2
Q/Q(M)-478096 Report a Problem



Triangular lunulae are:
1

Part of the syndrome caused by a LMX1b mutation
Triangular lunulae are part of the Nail-Patella syndrome. Findings in this syndrome include LMX1b
mutation, hypoplastic or absent patellae, nephropathy, a "Lester Iris" - hyperpigmentation of the pupillary
margin, cataracts, heterochromia irides and glaucoma. ATP2C1 and SERCA2 are associated with Hailey-
Hailey and Darier-White disease respectively.
Q/Q(M)-478096 Report a Problem
36



A child presents with sparse, fine hair, thin nails and loose skin in infancy. He has a pear-shaped, broad
nose and on X-ray has cone-shaped epiphyses. Which of the following genes is mutated in this syndrome?
1

P63
2

DLX3
3

RMRP
4

TRPS1
5

ABCC6
Q/Q(M)-478077 Report a Problem
A child presents with sparse, fine hair, thin nails and loose skin in infancy. He has a pear-shaped, broad
nose and on X-ray has cone-shaped epiphyses. Which of the following genes is mutated in this syndrome?
4

TRPS1
TRPS1 is the gene mutated in Tricho-rhino-phalangeal syndrome. TRPS1 is a putative zinc finger
transcription factor that is passed in an autosomal dominant fashion. P63 is mutated in Hay-Wells
syndrome and Ectrodactyly-ectodermal dysplasia-clefting (EEC) dyndrome. DLX3 (distal-less homeobox-
3) gene is associated with Trichodentoosseous syndrome. RMRP (RNA component of RNase MRP) is
defective in Cartilage-Hair hypoplasia syndrome. ABCC6 is defective in pseudoxanthoma elasticum.
Q/Q(M)-478077 Report a Problem


Fraying of the cuticles is a sign of which of the following conditions?
1

Dermatomyositis
2

Systemic sclerosis
3

Lupus erythematosus
4

Hypothyroidism
5

Lead poisoning
Q/Q(M)-474586 Report a Problem



Fraying of the cuticles is a sign of which of the following conditions?
1

Dermatomyositis
Dermatomyositis can be associated with fraying of the cuticles (Samitz sign).
Q/Q(M)-474586 Report a Problem



Which of the following is the predominant dermatoscopic finding seen in alopecia areata?
1

Diffuse white knots and a brush-pattern
2

Perifollicular arborizing vessels
3

Yellow dots
37

4

Reduction of follicular ostia
5

Small oval nodes
Q/Q(M)-482468 Report a Problem

Which of the following is the predominant dermatoscopic finding seen in alopecia areata?
3

Yellow dots
Dermatoscopic findings of alopecia areata include yellow dots, dystrophic hair shafts features, and
hypopigmented vellus hairs. Diffuse white knots and a brush pattern is seen in trichorrhexis nodosa. In
lichen planopilaris, a reduction to absence of follicular ostia, perifollicular scale and arborizing vessels,
pigmented networks, and white to blue-gray dots can be observed. Small oval nodes are seen in
monilethrix.
Q/Q(M)-482468 Report a Problem


Splinter hemorrhage of the nail can be seen with which of the following parasitic infections?
1

Scabies
2

Trichinosis
3

Sparganosis
4

Dracunculiasis
5

Gnathostomiasis
Q/Q(M)-474585 Report a Problem


Splinter hemorrhage of the nail can be seen with which of the following parasitic infections?
2

Trichinosis
Trichinosis can cause splinter hemorrhage of the nails.
Q/Q(M)-474585 Report a Problem


The hair finding characterized by an invagination of the distal hair shaft into the cup formed by the
proximal hair shaft is:
1

Trichoschisis
2

Pili torti
3

Pili triangulati
4

Trichorrhexis nodosa
5

Trichorrhexis invaginata
Q/Q(M)-478072 Report a Problem

The hair finding characterized by an invagination of the distal hair shaft into the cup formed by the
proximal hair shaft is:
5

Trichorrhexis invaginata
38

The above description is that of trichorrhexis invaginata, which is characteristic of Netherton syndrome.
Other findings include: ichthyosis linearis circumflexa and atopy. It is caused by the mutation of the serine
protease inhibitor, SPINK5.
Q/Q(M)-478072 Report a Problem


Which of the following syndromes does NOT have the finding shown in the image as part of the
spectrum?
1

Crandall
2

Argininosuccinic aciduria
3

Citrullinemia
4

Menke's Kinky Hair
5

Netherton
Q/Q(M)-478071 Report a Problem

Which of the following syndromes does NOT have the finding shown in the image as part of the
spectrum?
1

Crandall
Crandall syndrome has pili torti as its hair finding. The other options all have trichorrhexis nodosa as one
of the hair findings in the syndrome. Trichorrhexis nodosa is the fraying of hair ends so that the resemble
a "broom-stick" or "paint brush".
Q/Q(M)-478071 Report a Problem

A 7-year old girl presents to clinic with onychomadesis afflicting all nails. Her mother reports that she has
been healthy other than a self-limited illness two months prior. What is the most likely causative etiology:
1

Rheumatic fever
2

Kawasaki disease
3

Systemic T cell lymphoma
4

Yellow nail syndrome
5

Hand, Foot, and Mouth Disease
Q/Q(M)-482929 Report a Problem
39


A 7-year old girl presents to clinic with onychomadesis afflicting all nails. Her mother reports that she has
been healthy other than a self-limited illness two months prior. What is the most likely causative etiology:
5

Hand, Foot, and Mouth Disease
Coxsackievirus, the virus most associated with hand, foot, and mouth disease, is a well-known cause of
onychomadesis in children. Rheumatic fever has been associated with Mees lines of the nail. Yellow
nail syndrome is associated with yellow nail plates, edema, and lung disease. The other answer choices are
not commonly associated with nail disease.
Q/Q(M)-482929 Report a Problem

Which of the following nutritional deficiencies is associated with perifollicular hemorrhage?
1

Vitamin C
2

Niacin
3

Vitamin B12
4

Folate
5

Zinc
Q/Q(M)-482814 Report a Problem
Which of the following nutritional deficiencies is associated with perifollicular hemorrhage?
1

Vitamin C
Vitamin C deficiency, or scurvy, may lead to perifollicular hemorrhage. Niacin deficiency may result in a
photo-distributed eruption, diarrhea, and psychiatric symptoms (diarrhea, dermatitis, and dementia).
Vitamin B12 may lead to glossitis with a shiny, painful tongue. Zinc deficiency may result in a
periorificial dermatitis.
Q/Q(M)-482814 Report a Problem



A 24-year-old man with Lesch-Nyhan syndrome is most likely to present with what nail disorder:
1

Beaus lines
2

Mees lines
3

Muehrckes lines
4

Onychophagia
5

Dolichonychia
Q/Q(M)-482948 Report a Problem



A 24-year-old man with Lesch-Nyhan syndrome is most likely to present with what nail disorder:
4

Onychophagia
Oncyhophagia refers to nail biting. Patients with Lesch-Nyhan syndrome, caused by defective HGPRT
enzyme, are prone to self-mutilating behaviors, including onychophagia. Beaus lines are transverse
indented nail plate furrows caused by temporary growth arrest of the nail matrix, often due to
40

chemotherapy or other stressful events and illnesses. Mees lines are transverse white lines affecting all
nails and growing out with the nail; there is no associated indentation. Mees lines may be caused by
arsenic poisoning, rheumatic fever, or other systemic diseases. Muehrckes lines disappear with
squeezing of the nail and are due to disorders of low albumin. Dolichonychia exhibits long, slender nails,
and is associated with Ehlers-Danlos and Marfans.
Q/Q(M)-482948 Report a Problem

Distal subungual onycholysis associated with paronychia is due to:
1

Candida spp
2

Fusarium spp
3

Scytalidium dimidiatum
4

Trichophyton rubrum
5

Trichophyton mentagrophytes
Q/Q(M)-475892 Report a Problem



Distal subungual onycholysis associated with paronychia is due to:
3

Scytalidium dimidiatum
This mold is the only one responsible for paronychia in DLSO and is more often observed in finger than in
toenails.
Q/Q(M)-475892 Report a Problem



Which of the following are findings associated with Nail-Patella Syndrome?
1

Glomerulonephritis
2

Alopecia
3

Bladder diverticuli
4

Photosensitivity
5

Hip dysplasia
Q/Q(M)-477403 Report a Problem


Which of the following are findings associated with Nail-Patella Syndrome?
1

Glomerulonephritis
Nail patella syndrome is a rare, AD syndrome associated with nail hypoplasia, absent or hypoplastic
patella, posterior iliac horns, glaucoma, hyperpigmentation of the pupillary margin or Lester iris, and
glomerulonephritis and nephrotic syndrome. Glomerulonephritis and renal failure are features seen in nail
patella syndrome. This syndrome is also known as HOOD syndrome or hereditary osteoonychodysplasia.
Q/Q(M)-477403 Report a Problem



41

Which of the following is NOT a cause of red lunulae?
1

Endocarditis
2

Psoriasis
3

Carbon monoxide poisoning
4

Imuran
5

Rheumatoid arthritis
Q/Q(M)-478120 Report a Problem


Which of the following is NOT a cause of red lunulae?
1

Endocarditis
Endocarditis is not a cause of red lunulae. Alopecia areata, psoriasis, prednisone for connective tissue
diseases, carbon monoxide poisoning, cardiac failure, SLE, RA, COPD, CO2, cirrhosis, imuran and
radiation are the causes of this condition.
Q/Q(M)-478120 Report a Problem



Which of the following conditions with hair abnormalities is caused by mutations in a DNA helicases?
1

Anhidrotic Ectodermal Dysplasia
2

Hidrotic Ectodermal Dysplasia
3

KID Syndrome
4

Rothmund-Thomson Syndrome
5

Incontinentia Pigmenti
Q/Q(M)-474561 Report a Problem


Which of the following conditions with hair abnormalities is caused by mutations in a DNA helicases?
4

Rothmund-Thomson Syndrome
Rothmund-Thopmson syndrome is casued by mutations in RecQL4. Key clinical features include
poikiloderma, photosensitivity, dystrophic nails, hypoplastic thumbs, cataracts, and reports of
osteosarcoma. Anhidrotic ectodermal dysplasia is associated with mutations in ectodysplasin. Hidrotic
ectodermal dysplasia is associated with mutations in connexin 30. Kid syndrome is associated with
mutations in connexin 26 and IP with NEMO.
Q/Q(M)-474561 Report a Problem


A 16-year-old woman presents with hair breakage. Examination of a single shaft reveals division into two
shafts at the distal-most portion of the hair. What is the most likely diagnosis:
1

Pili multigemini
2

Pili bifurcati
3

Trichoptilosis
4

Trichoschisis
42

5

Trichothiodystrophy
Q/Q(M)-482943 Report a Problem


A 16-year-old woman presents with hair breakage. Examination of a single shaft reveals division into two
shafts at the distal-most portion of the hair. What is the most likely diagnosis:
3

Trichoptilosis
Pili multigemini refers to a rare disorder of multiple divided hair matrices, with multiple hair shafts each
with their own cuticle all arising from one follicle. Pili bifucarti exhibits hair shaft bifurcation into short
segments along hair shafts, with each segment covered with its own cuticle. Trichoptilosis refers to
split ends of the distal portion of the hair shaft. Trichoschisis is a clean fracture of the hair shaft.
Trichothiodystrophy is a disorder of beaded hair when viewed with polarizing light.
Q/Q(M)-482943 Report a Problem



Which part of the hair follicle is the first to cornify?
1

Huxleys layer of inner root sheath
2

Henleys layer of inner root sheath
3

Outer root sheath
4

Medulla
5

Cuticle
Q/Q(M)-482921 Report a Problem


Which part of the hair follicle is the first to cornify?
2

Henleys layer of inner root sheath
Henleys layer of inner root sheath. Henleys layer is the outer layer of the inner root sheath. The inner
root sheath is keratinized with trichohyaline granules and is shed with the growing hair shaft at the level of
the isthmus
Q/Q(M)-482921 Report a Problem


A defect in which part of the nail cause a true leukonychia?
1

Cuticle
2

Hyponychium
3

Lunula
4

Nail bed
5

Nail plate
Q/Q(M)-476521 Report a Problem


A defect in which part of the nail cause a true leukonychia?
5

Nail plate
43

True leukonychia is caused by an acute stress to the nail and manifests in the nail plate. Examples include
Terry's nails and Mee's lines. Apparent leukonychia, on the other hand, are a result of problems in the nail
bed and are indicative of overall health compromise. Examples of apparent leukonychia include Lindsay's
and Muehrcke's nail.
Q/Q(M)-476521 Report a Problem


Ventral pterygium pictured below is characteristically associated with what disorder?
1

Lichen planus
2

Systemic sclerosis
3

Cicatricial pemphigoid
4

SJS/TEN
5

Chronic GVHD
Q/Q(M)-475941 Report a Problem


Ventral pterygium pictured below is characteristically associated with what disorder?
2

Systemic sclerosis
Ventral pterygium is most characteristically associated with systemic sclerosis. All other choices are
associated with dorsal pterygium.
Q/Q(M)-475941 Report a Problem


A patient presents with 20 nails with absent cuticles and lunulae, slow growth, dystrophic shape and a
yellow hue. Which of the following findings is/are associated?
1

Upper extremity edema
2

Cirrhosis
3

Pleural effusions
4

Chest pain
5

Pulmonary fibrosis
Q/Q(M)-478095 Report a Problem



44

A patient presents with 20 nails with absent cuticles and lunulae, slow growth, dystrophic shape and a
yellow hue. Which of the following findings is/are associated?
3

Pleural effusions
Yellow Nail syndrome has been associated with lower extremity edema, bronchiectasis and pleural
effusions. The other listed findings are not part of Yellow Nail syndrome.
Q/Q(M)-478095 Report a Problem


Papular atrichia is caused by mutations in which gene?
1

Wingless
2

Patched
3

Hairless
4

Fox forehead
5

Distal-less homebobox
Q/Q(M)-474570 Report a Problem


Papular atrichia is caused by mutations in which gene?
3

Hairless
Hairless gene mutations cause papular atrichia.
Q/Q(M)-474570 Report a Problem



A 66-year-old woman presents with black heads on the nose. Examination reveals multiple small
hairs protruding from the same follicle. What is the most likely associated diagnosis:
1

Pili multigemini
2

Pili bifurcati
3

Trichostasis spinulosa
4

Trichoschisis
5

Trichothiodystrophy
Q/Q(M)-482944 Report a Problem


A 66-year-old woman presents with black heads on the nose. Examination reveals multiple small
hairs protruding from the same follicle. What is the most likely associated diagnosis:
3

Trichostasis spinulosa
Pili multigemini refers to a rare disorder of multiple divided hair matrices, with multiple hair shafts each
with their own cuticle all arising from one follicle. Pili bifucarti exhibits hair shaft bifurcation into short
segments along hair shafts, with each segment covered with its own cuticle. Trichostasis spinulosa
refers to follicles filled with bundles of vellus hairs. This disorder may be associated with renal failure.
Trichoschisis is a clean fracture of the hair shaft. Trichothiodystrophy is a disorder of beaded hair when
viewed with polarizing light.
Q/Q(M)-482944 Report a Problem
45



A 34-year-old man undergoing chemotherapy for acute myelogenous leukemia presents with smooth
white horizontal bands involving the nail plates of all digits. What is the most likely disorder:
1

Beaus lines
2

Mees lines
3

Muehrckes lines
4

Onychophagia
5

Dolichonychia
Q/Q(M)-482946 Report a Problem


A 34-year-old man undergoing chemotherapy for acute myelogenous leukemia presents with smooth
white horizontal bands involving the nail plates of all digits. What is the most likely disorder:
2

Mees lines
Beaus lines are transverse indented nail plate furrows caused by temporary growth arrest of the nail
matrix, often due to chemotherapy or other stressful events and illnesses. Mees lines are transverse
white lines affecting all nails and growing out with the nail; there is no associated indentation. Mees
lines may be caused by chemotherapy, arsenic poisoning, rheumatic fever, or other systemic diseases.
Muehrckes lines disappear with squeezing of the nail and are due to disorders of low albumin.
Oncyhophagia refers to nail biting. Dolichonychia exhibits long, slender nails, and is associated with
Ehlers-Danlos and Marfans.
Q/Q(M)-482946 Report a Problem

Chronic paronychia is most commonly caused by which of the following organisms?
1

Candida albicans
2

Candida tropicalis
3

Pseudomonas aeruginosa
4

Staphylococcus aureus
5

Trichophyton rubrum
Q/Q(M)-474581 Report a Problem


Chronic paronychia is most commonly caused by which of the following organisms?
1

Candida albicans
Candida albicans is the most common pathogen associated with chronic paronychia.
Q/Q(M)-474581 Report a Problem


Which part of the matrix gives rise to the dorsal nail plate?
1

Distal matrix
2

Proximal matrix
3

Ventral matrix
46

4

Dorsal matrix
5

Lateral matrix
Q/Q(M)-474575 Report a Problem


Which part of the matrix gives rise to the dorsal nail plate?
2

Proximal matrix
The proximal matrix gives rise to the dorsal nail plate, while the distal matrix gives rise to the ventral nail
plate.
Q/Q(M)-474575 Report a Problem



Regarding androgens in women, which of the following statements is NOT correct?
1

Testosterone binds the androgen receptor
2

Eyebrows, eyelashes and vellus hairs are androgen-dependent
3

The hair follicle requires conversion of testosterone to dihydrotestosterone for expression of
androgen action
4

Dihydrotestosterone binds the androgen receptor
5

There are no differences in eyelashes, eyebrows and vellus hair-bearing areas in men and women
Q/Q(M)-478080 Report a Problem

Regarding androgens in women, which of the following statements is NOT correct?
2

Eyebrows, eyelashes and vellus hairs are androgen-dependent
Only testosterone and dihydrotestosterone bind the androgen receptor, Eyebrows, eyelashes and vellus
hairs are NOT androgen-dependent, thus there are no differences between these areas in men and women,
and the hair follicle requires conversion of testosterone to dihydrotestosterone for expression of androgen
action.
Q/Q(M)-478080 Report a Problem


Yellow nails can be seen in each of the following syndromes except:
1

Bronchiectasis
2

Hyperlipidemia
3

Tobacco use
4

Thyroid disease
5

Penicillamine
Q/Q(M)-477680 Report a Problem

Yellow nails can be seen in each of the following syndromes except:
2

Hyperlipidemia
Yellow discoloration of the nail plate can be associated with bronchiectasis or underlying pulmonary
47

disease such as COPD. Other associated conditions include lymphedema, rheumatoid arthritis, thyroid
disease, malignancies, penicillamine, and chronic sinusitis or bronchitis. Hyperlipidemia is not associated
with nail changes.
Q/Q(M)-477680 Report a Problem


Fungal finger onycholysis usually results from:
1

Trichophyton rubrum
2

Trichophyton interdigitale
3

Candida albicans
4

Scopulariopsis brevicaulis
5

Fusarium spp
Q/Q(M)-475891 Report a Problem

Fungal finger onycholysis usually results from:
3

Candida albicans
It is widely accepted that C. albicans acts as colonizer that has found an ideal environment.
Q/Q(M)-475891 Report a Problem



How long do hairs stay in telogen before they are released?
1

3-4 weeks
2

3-4 months
3

3-4 years
4

3-4 days
5

3-4 hours
Q/Q(M)-482932 Report a Problem


How long do hairs stay in telogen before they are released?
2

3-4 months
Hairs remain in telogen phase for 3-4 months before they are released. The length of anagen phase varies
for hairs of different parts of the body and determines the ultimate length of the hair. Scalp hair may
remain in anagen for 3-5 years. The duration of catagen phase is typically 3-4 weeks.
Q/Q(M)-482932 Report a Problem

Arsenical poisoning is associated with what nail finding?
1

Muehrcke's lines
2

Mee's lines
3

Hapalonychia
48

4

Onycholysis
5

Oil spots
Q/Q(M)-474577 Report a Problem


Arsenical poisoning is associated with what nail finding?
2

Mee's lines
Arsenical poisoning is characteristically associated with Mee's lines.
Q/Q(M)-474577 Report a Problem



Scalp biopsy of which of the following types of hair loss will reveal increased telogen hairs?
1

Lichen planopilaris
2

Discoid lupus erythematosis
3

Central centrifugal alopecia
4

Dissecting cellulitis
5

Androgenetic alopecia
Q/Q(M)-482936 Report a Problem

Scalp biopsy of which of the following types of hair loss will reveal increased telogen hairs?
5

Androgenetic alopecia
Androgenetic alopecia, telogen effluvium, and alopecia areata all have an increased fraction of telogen
hairs. Lichen planopilaris and discoid lupus typically reveal a chronic inflammatory alopecia on biopsy.
Dissecting cellulitis typically exhibits chronic inflammation with hypertrophic scar tissue intervening
between suppurative follicles.
Q/Q(M)-482936 Report a Problem



Yellow nail syndrome is associated with which of the following?
1

Multiple myeloma
2

Panhypopituitarism
3

Diabetes mellitus
4

Dermatophyte infection
5

Lymphedema and bronchiectasis
Q/Q(M)-474584 Report a Problem


Yellow nail syndrome is associated with which of the following?
5

Lymphedema and bronchiectasis
Lymphedema and bronchiectasis are associated with yellow nail syndrome.
49

Q/Q(M)-474584 Report a Problem


Pili Annulati is associated with which of the following syndromes?
1

Netherton
2

PIBIDS
3

Citrullinemia
4

Hidrotic ectodermal dysplasia
5

Alopecia areata
Q/Q(M)-478074 Report a Problem


Pili Annulati is associated with which of the following syndromes?
5

Alopecia areata
Pili Annulati is the alternating light and dark bands secondary to air-spaces seen in normal light and has
been associated with Alopecia areata. The "tiger-tail" pattern seen in trichothiodystrophy (PIBIDS) is only
seen with polarized light. Netherton and Citrullinemia do not have this hair finding associated.
Q/Q(M)-478074 Report a Problem



Blue lunulae are associated with each of the following except:
1

Argyria
2

Cardiac failure
3

Quinacrine
4

Wilsons disease
5

Phenolphthalein
Q/Q(M)-477187 Report a Problem


Blue lunulae are associated with each of the following except:
2

Cardiac failure
Blue lunulae have been reported in association with argyria, Wilsons disease, hereditary acrolabial
telangectasia, paronychia, phenolphthalein, quinacrine, topical bichloride, and mercury exposure.
Q/Q(M)-477187 Report a Problem


Signs of virilization include all of the following EXCEPT:
1

Centripetal obesity
2

Temporal balding
3

Clitoromegaly
4

Increased muscle mass in the limb girdles
5

Acne
50

Q/Q(M)-478085 Report a Problem

Signs of virilization include all of the following EXCEPT:
1

Centripetal obesity
Centripetal obesity is a sign of cortisol excess not of virilization. In addition to the those listed above,
irregular or absent menses and deepening of the voice are signs of virilization.
Q/Q(M)-478085 Report a Problem

The best diagnosis for this scarring form of alopecia is
1

Chronic cutaneous lupus erythrematosus
2

Lichen planopilaris
3

Peseudopelade of Broq
4

Perifolliculitis capitis abscedens et suffodiens
5

Central centrifugal cicatricial alopecia
Q/Q(M)-482127 Report a Problem


The best diagnosis for this scarring form of alopecia is
4

Perifolliculitis capitis abscedens et suffodiens
Perifolliculitis capitis abscedens et suffodiens is also known as dissecting cellulitis of scalp is uncommon
chronic suppurative disorder that most often affects young black men. Together with acne conglobata,
hidradenitis suppurativa and pilonidal cysts, it is a component of the 'follicular occlusion tetrad'. Patients
present with multiple painful inflammatory nodules and fluctuant abscesses of the scalp, which are often
interconnected via sinus tracts. The vertex and occiput are the sites of predilection. As the disease
progresses, scarring alopecia may develop. All other choices can cause scarring alopecia but with different
clinical picture.
Q/Q(M)-482127 Report a Problem


A 68-year old man presents with double white transverse lines on all nails that disappear with
compression of the nail plate. What is the most likely associated systemic disease:
1

Ehlers-Danlos Syndrome
2

Arsenic poisoning
3

Rheumatic Fever
4

Neurofibromatosis
51

5

Nephrotic Syndrome
Q/Q(M)-482928 Report a Problem


A 68-year old man presents with double white transverse lines on all nails that disappear with
compression of the nail plate. What is the most likely associated systemic disease:
5

Nephrotic Syndrome
Muehrckes lines are white transverse lines from an abnormal vascular bed. They disappear with
pressure on the nail plate. Muehrckes lines are associated with disorders causing low albumin, such as
nephrotic syndrome, liver disease, and malnutrition.
Q/Q(M)-482928 Report a Problem



Transverse white bands on one or two nails is caused by:
1

Trauma to the matrix
2

Alopecia areata
3

Psoriasis
4

Systemic lupus erythematosis
5

Lichen planus
Q/Q(M)-478097 Report a Problem


Transverse white bands on one or two nails is caused by:
1

Trauma to the matrix
Longitudinal leukonychia or transverse white bands are caused by trauma to the nail matrix. AA,
psoriasis, SLE and LP all cause spotted red lunulae, not transverse white bands.
Q/Q(M)-478097 Report a Problem

A patient with curly hair that straightened after puberty, enamel hypoplasia, dental pits, and increased
bone density likely has a mutation in which of the following genes?
1

Distal-less homeobox-3 gene (DLX3)
2

Vascular-endothelial growth factor receptor 3
3

Bone morphogenetic protein type 2
4

SLURP 1
5

SPINK 5
Q/Q(M)-474559 Report a Problem



A patient with curly hair that straightened after puberty, enamel hypoplasia, dental pits, and increased
bone density likely has a mutation in which of the following genes?
1

Distal-less homeobox-3 gene (DLX3)
52

Distal-less homeobox-3 gene (DLX3) mutations cause trichodentoosseous syndrome, which is
characterized by curly hair that straightens in the 2nd-3rd decades, enamel hypoplasia, dental pits, and
increased bone density.
Q/Q(M)-474559 Report a Problem



A 24-year-old man reports that he has developed white patches of scalp hair that seem to migrate across
his scalp. Examination reveals three 1cm foci of poliosis with surrounding normally pigmented hair. He
has a history of hypothyroidism, but no other systemic diseases. What is the most likely etiology:
1

Form fruste of alopecia areata
2

Vogt-Koyanagi-Harada syndrome
3

Tinea capitis
4

Allezandrini syndrome
5

Lichen planopilaris
Q/Q(M)-482933 Report a Problem


A 24-year-old man reports that he has developed white patches of scalp hair that seem to migrate across
his scalp. Examination reveals three 1cm foci of poliosis with surrounding normally pigmented hair. He
has a history of hypothyroidism, but no other systemic diseases. What is the most likely etiology:
1

Form fruste of alopecia areata
Forme fruste, or limited, alopecia areata, may present with migratory poliosis due to destruction of
follicular melanocytes but survival of the hair-producing bulb. Vogt-Koyanagi-Harada syndrome presents
as meningoencephalitis followed by visual and hearing deficits and poliosis of the periorbital region. The
other answer choices are not typically associated with migratory poliosis.
Q/Q(M)-482933 Report a Problem


This hair disorder and mutations in ATP7A (MNK) gene are found in what genodermatosis?
1

Bjornstad syndrome
2

Menke's kinky hair syndrome
3

Crandall's syndrome
4

Bazex's follicular atrophoderma
5

Trichothiodystrophy
Q/Q(M)-475929 Report a Problem
53




This hair disorder and mutations in ATP7A (MNK) gene are found in what genodermatosis?
2

Menke's kinky hair syndrome
Menke's kinky hair syndrome is characterized by pili torti and mutations in APT7A (MNK) gene.
Bjornstad syndrome, Crandalls syndrome and Bazexs follicular atrophoderma are associated with pili
torti but are not associated with this mutation.
Q/Q(M)-475929 Report a Problem



Atrophy of the isthmus is a characteristic histologic finding of which of the following conditions?
1

Discoid lupus erythematosus
2

Lichen planopilaris
3

Parry-Romberg syndrome
4

Folliculitis Decalvans
5

Acne Keloidalis Nuchae
Q/Q(M)-474571 Report a Problem



Atrophy of the isthmus is a characteristic histologic finding of which of the following conditions?
2

Lichen planopilaris
Lichen planopilaris is characterized by perifollicular lymphocytic infiltration with atrophy of the isthmus.
Q/Q(M)-474571 Report a Problem


Familial alopecia areata is commonly associated with what laboratory abnormality?
1

Hypocalcemia
2

Hypereosinophilia
3

Hyperkalemia
4

Leukopenia
5

Thrombocytopenia
Q/Q(M)-482934 Report a Problem
Familial alopecia areata is commonly associated with what laboratory abnormality?
54

5

Thrombocytopenia
Familial alopecia areata has been associated with persistent thrombocytopenia. Alopecia areata has
recently been linked to the ULBP3 gene. Alopecia areata is associated with other autoimmune disorders
such as thyroid disease and vitiligo.
Q/Q(M)-482934 Report a Problem



What condition most likely describes a hair defect with ruffled cuticle in 4-year old female with blond
hair?
1

Pili torti
2

Pili annulati
3

Wooly hair
4

Loose anagen hair syndrome
5

Monilethrix
Q/Q(M)-476556 Report a Problem


What condition most likely describes a hair defect with ruffled cuticle in 4-year old female with blond
hair?
4

Loose anagen hair syndrome
Loose anagen hair syndrom occurs primarily in young, females with blond hair. The defect results in early
keratinization of the inner root sheath which leads to poor anchoring. Examination of hair shaft reveals
ruffled proximal cuticle.
Q/Q(M)-476556 Report a Problem



A patient with diffuse hair loss developing after a thallium scan likely has which of the following
conditions?
1

Anagen effluvium
2

Catagen effluvium
3

Telogen effluvium
4

Loose anagen syndrome
5

Uncombable hair syndrome
Q/Q(M)-474568 Report a Problem



A patient with diffuse hair loss developing after a thallium scan likely has which of the following
conditions?
1

Anagen effluvium
Anagen effluvium can result from infusions of thallium.
Q/Q(M)-474568 Report a Problem
55




Argininosuccinic aciduria is characteristically associated with this hair finding name the condition
pictured below:
1

Pili trianguli et canaliculi
2

Trichoschisis
3

Monilethrix
4

Trichorrhexis nodosa
5

Trichorrhexis invaginata
Q/Q(M)-475930 Report a Problem

Argininosuccinic aciduria is characteristically associated with this hair finding name the condition
pictured below:
4

Trichorrhexis nodosa
Trichorrhexis nodosa is a finding in argininosuccinic aciduria.
Q/Q(M)-475930 Report a Problem


A 36-year-old woman presents with arthralgias and three fingernails exhibiting adherence of the ventral
surface of the distal nail plate to the hyponychium. What is the most likely associated disease:
1

Psoriasis
2

Lichen Planus
3

Systemic Sclerosis
4

Yellow Nail Syndrome
5

Multicentric Reticulohistiocytosis
Q/Q(M)-482931 Report a Problem


A 36-year-old woman presents with arthralgias and three fingernails exhibiting adherence of the ventral
surface of the distal nail plate to the hyponychium. What is the most likely associated disease:
3

Systemic Sclerosis
The most common cause of ventral pterygium is systemic sclerosis, which often first presents with
56

arthralgias. Lichen planus is most commonly associated with dorsal, rather than ventral, pterygium.
Multicentric reticulohistiocytosis is associated with deforming arthritis and coral bead nodules around
the proximal nail fold, but not with pterygium.
Q/Q(M)-482931 Report a Problem


The common culprit of proximal white subungual onychomycosis is :
1

Candida albicans
2

Trichophyton rubrum
3

Trichophyton interdigitale
4

Scydalidium dimidiatum
5

Fusarium spp
Q/Q(M)-475894 Report a Problem


The common culprit of proximal white subungual onychomycosis is :
2

Trichophyton rubrum
T. rubrum is responsible for more than 95% of these cases.
Q/Q(M)-475894 Report a Problem


Highest graft survival in hair transplantation is achieved through the use of:
1

4-5 mm plugs
2

Minigrafts
3

Single hairs
4

Follicular units
5

Micrografts
Q/Q(M)-477177 Report a Problem


Highest graft survival in hair transplantation is achieved through the use of:
4

Follicular units
Seager et. al examined the survival rate of single hair grafts and compared them to follicular unit grafts in
a single patient. They found that there was a much higher survival rate in the follicular unit grafts. They
hypothesized that the extra tissue surrounding the follicular unit grafts protected them from crush injury.
Q/Q(M)-477177 Report a Problem


Lindsay's nails (distal nail normal, proximal nail white) is characteristic of:
1

Chronic renal failure
2

Plummer-Vinson syndrome
3

Hemochromatosis
4

Ectodermal dysplasia
57

5

LEOPARD syndrome
Q/Q(M)-478124 Report a Problem

Lindsay's nails (distal nail normal, proximal nail white) is characteristic of:
1

Chronic renal failure
Lindsay's nails are characteristic of chronic renal failure and indicate nailbed edema. The remaining
options are causes of koilonychia, a spooning defect of the nail.
Q/Q(M)-478124 Report a Problem

Which of the following syndromes can present with woolly or curly hair?
1

Bjornstad syndrome
2

Citrullinemia
3

Menkes syndrome
4

Noonan syndrome
5

Netherton syndrome
Q/Q(M)-482508 Report a Problem


Which of the following syndromes can present with woolly or curly hair?
4

Noonan syndrome
Noonan syndrome, Naxos syndrome, Carvajal syndrome, Costello syndrome, and Trichodentoosseous
syndrome can present with curly or woolly hair. Bjornstad syndrome and Menkes syndrome present with
pili torti. Citrullinemia presents with trichorrhexis nodosa.
Q/Q(M)-482508 Report a Problem



Pathognomonic nails changes in HOOD syndrome or Hereditary osteoonychodysplasia are:
1

Red and white longitudinal banding
2

Pterygium inversum
3

Yellow nails
4

Pincer nails
5

Triangular lunulae
Q/Q(M)-477649 Report a Problem

Pathognomonic nails changes in HOOD syndrome or Hereditary osteoonychodysplasia are:
5

Triangular lunulae
In the HOOD syndrome (Hereditary osteoonychodysplasia) or the Nail-Patella syndrome, patients have
different degrees of nail dysplasia. In general, the thumbs are most severely involved and nail disease
decreases in severity from the second to the fifth digits. Characteristic nail changes include longitudinal
ridging, koilonychia, and splitting. Toenails are rarely involved. Pathognomonic nail change is the V
58

shaped triangular lunulae with the distal peak in the midline.
Q/Q(M)-477649 Report a Problem


Choose the correct statement regarding white piedra.
1

Commonly occurs in the tropics
2

Consists of cream-colored colonies attached to hairs
3

Treatment consists of cutting the hair
4

Is caused by piedra hortae
5

Both Consists of cream-colored colonies attached to hairs and Treatment consists of cutting the hair
Q/Q(M)-478744 Report a Problem


Choose the correct statement regarding white piedra.
5

Both Consists of cream-colored colonies attached to hairs and Treatment consists of cutting the
hair
White piedra: caused by Trichosporon beiglii, occurs commonly in temperate climates. The culture shows
cream colored, colonies attached to hairs. Treatment consists of cutting hair. Black piedra is caused by
piedra hortae.
Q/Q(M)-478744 Report a Problem


An 84-year-old man presents to clinic with atypical fingernails and is diagnosed with hapalonychia. What
defect does this term represent?
1

Long nails
2

White nail plates
3

Edema of the nail bed
4

Broad nails
5

Soft nails
Q/Q(M)-482927 Report a Problem


An 84-year-old man presents to clinic with atypical fingernails and is diagnosed with hapalonychia. What
defect does this term represent?
5

Soft nails
Hapalonychia refers to soft nails and is caused by a defect in the nail matrix. Dolichonychia describes
abnormally long, narrow nails, while brachyonchia describes abnormally short, broad nails. Edema of the
nail bed may present as Meurhckes lines.
Q/Q(M)-482927 Report a Problem


Which palmoplantar keratoderma is characterized by painful hyperkeratosis?
1

Richner-Hanhart
2

Howell-Evans
59

3

Vohwinkle
4

Unna-thost
5

Mal de Meleda
Q/Q(M)-482862 Report a Problem

Which palmoplantar keratoderma is characterized by painful hyperkeratosis?
1

Richner-Hanhart
While most palmoplantar keratodermas are characterized by thick plaques of the palms and soles that are
usually painless, Richner Hanhart is classically associated with painful lesions, rendering it distinctive.
Richner Hanhart is also associated with a painful keratitis and pigment dilution of the hair.
Q/Q(M)-482862 Report a Problem


What is the mode of inheritance of this disorder of the hair shaft?
1

Autosomal dominant
2

Autosomal recessive
3

X-linked dominant
4

X-linked recessive
5

Sporadic
Q/Q(M)-476507 Report a Problem



What is the mode of inheritance of this disorder of the hair shaft?
1

Autosomal dominant
Monilethrix is an autosomal dominant condition with variable penetrance. The hair shaft has an elliptical
or beaded hair appearance.
Q/Q(M)-476507 Report a Problem


Tays syndrome is characterized by this hair condition pictured below identify the condition:
1

Trichorrhexis nodosa
2

Trichorrhexis invaginata
3

Trichoschisis
4

Monilethrix
60

5

Pili trianguli et canaliculi
Q/Q(M)-475931 Report a Problem


Tays syndrome is characterized by this hair condition pictured below identify the condition:
3

Trichoschisis
Tays syndrome is also known as trichothiodystrophy which is characterized by trichoschisis or clean
fracture of the hair shaft.
Q/Q(M)-475931 Report a Problem


A six year old girl presents after being sent home from school for "lice". On examination, there are
structures on the hair which are freely slide along the hair shaft. What is the most likely diagnosis?
1

Pediculosis
2

Hair casts
3

Trichoptilosis
4

Monilithrix
5

Pili annulati
Q/Q(M)-478075 Report a Problem


A six year old girl presents after being sent home from school for "lice". On examination, there are
structures on the hair which are freely slide along the hair shaft. What is the most likely diagnosis?
2

Hair casts
Hair casts present most commonly in young girls between the age of 2 and 8 years old. These casts are
freely sliding along the hair shaft and are not adherent. They must be distinguished from nits from
pediculosis capitis which are adherent to the hair and do not slide. Trichoptilosis are "spit ends" that form
from chemical or physical damage to the hair.
Q/Q(M)-478075 Report a Problem


A 52 year old male with history of diabetes mellitus with a >3 month history of recurrent warmth,
erythema, and slight pain surrounding the first toenail of the right foot. What is the most likely diagnosis?
1

Acute Paronychia
2

Orf
61

3

Contact Dermatitis
4

Chronic Paronychia
5

Herpetic Whitlow
Q/Q(M)-482889 Report a Problem

A 52 year old male with history of diabetes mellitus with a >3 month history of recurrent warmth,
erythema, and slight pain surrounding the first toenail of the right foot. What is the most likely diagnosis?
4

Chronic Paronychia
Paronychia is an inflammatory reaction involving the folds of skin surrounding the nail. It can be
characterized as ACUTE or CHRONIC -Acute paronychia usually due to direct or indirect trauma
followed by separation of eponychium from the nail plate, the causative bacteria usually Staph aureus and
Strep pyogenes. The treatment includes culturing the skin, systemic antibiotics, and draining of abscess if
necessary. -Chronic paranochya is most likely a contact reaction to irritants or allergens. It is characterized
by inflammation of the proximal nail fold with erythema, edema and absence of the cuticle. The most
frequently recovered organism is CANDIDA. It was originally thought to be result of Candida infection
and treatment was based upon antifungal therapy with minimal-moderate response. Newer studies suggest
that it is actually dermatitis of the nail fold COLONIZED by Candida. Treatment now is based on topical
anti-inflammatory steroids
Q/Q(M)-482889 Report a Problem


A 4 year-old boy has a single circular alopecic atrophic patch on the vertex of the scalp. This most likely
is due to which of the following conditions?
1

Aplasia cutis congenita
2

Congenital syphilis
3

Keratosis Follicularis Spinulosa Decalvans
4

Morphea en coup de sabre
5

Triangular Alopecia
Q/Q(M)-474573 Report a Problem


A 4 year-old boy has a single circular alopecic atrophic patch on the vertex of the scalp. This most likely
is due to which of the following conditions?
1

Aplasia cutis congenita
Aplasia cutis congenita often affects the vertex of the scalp and heals with a circular atrophic alopecic
scar.
Q/Q(M)-474573 Report a Problem


Psoriatic onycholysis is caused by psoriasis in which of the following nail subunits?
1

Nail matrix
2

Nail plate
3

Nail bed
4

Proximal nail fold
62

5

Hyponychium
Q/Q(M)-474582 Report a Problem


Psoriatic onycholysis is caused by psoriasis in which of the following nail subunits?
3

Nail bed
Nail bed involvement by psoriasis often causes onycholysis.
Q/Q(M)-474582 Report a Problem

This hair finding is caused by mutations in what gene?
1

SPINK 5
2

NEMO
3

SERCA2
4

Dyskerin
5

Keratin 16
Q/Q(M)-475932 Report a Problem



This hair finding is caused by mutations in what gene?
1

SPINK 5
Mutations in SPINK 5, a serine protease inhibitor leads to Nethertons syndrome which is characterized
by trichorrhexis invaginata or bamboo hair.
Q/Q(M)-475932 Report a Problem


Steatocystoma multiplex and natal teeth are associated with which of the following conditions?
1

Jadassohn-Lewandowsky pachyonychia congenita (Type 1)
2

Jackson-Sertole pachyonychia congenita (Type 2)
3

Nail-patella syndrome
4

Coffin-Siris syndrome
5

Rubenstein-Taybi syndrome
63

Q/Q(M)-474579 Report a Problem



Steatocystoma multiplex and natal teeth are associated with which of the following conditions?
2

Jackson-Sertole pachyonychia congenita (Type 2)
Jackson-Sertole pachyonychia congenita (Type 2) is characterized by thickening of the nailbed and plate,
steatocystoma multiplex, and natal teeth.
Q/Q(M)-474579 Report a Problem



Air spaces in the hair shaft lead to this condition pictured below name the condition:
1

Pili trianguli et canaliculi
2

Pili annulati
3

Trichoschisis
4

Pohl Pinkus constriction
5

Wooly hair
Q/Q(M)-475934 Report a Problem


Air spaces in the hair shaft lead to this condition pictured below name the condition:
2

Pili annulati
Pili anulati is caused by air spaces in the hair shaft.
Q/Q(M)-475934 Report a Problem


Bjrnstad syndrome is the combination of pili torti and which of the following features?
1

Lentigines
2

White forelock
3

Hypogonadism
4

Deafness
5

Seizures
Q/Q(M)-474551 Report a Problem


Bjrnstad syndrome is the combination of pili torti and which of the following features?
4

Deafness
64

Deafness and pili torti are the two cardinal features of Bjrnstad syndrome.
Q/Q(M)-474551 Report a Problem


Deposition of mucin in the hair follicle can be associated with which disease?
1

Keratosis Follicularis Spinulosa Decalvans
2

Adams-Oliver Syndrome
3

Mycosis fungoides
4

Acne Keloidalis Nuchae
5

Perifolliculitis Capitis Abscedens et Suffodiens
Q/Q(M)-474572 Report a Problem

Deposition of mucin in the hair follicle can be associated with which disease?
3

Mycosis fungoides
Mycosis fungoides can be associated with follicular mucinosis.
Q/Q(M)-474572 Report a Problem


Mutations in hair keratins hHB6 and hHB1 cause this characteristic finding pictured below name the
condition:
1

Trichoptilosis
2

Hair casts
3

Tiger-tail hair
4

Trichoschisis
5

Monilethrix
Q/Q(M)-475933 Report a Problem


Mutations in hair keratins hHB6 and hHB1 cause this characteristic finding pictured below name the
condition:
5

Monilethrix
Mutations in the hair keratins, hHB6 and hHB 1 cause monilethrix.
Q/Q(M)-475933 Report a Problem


Plummer's nails are associated with which of the following disorders?
65

1

Dermatomyositis
2

Systemic sclerosis
3

Lupus erythematosus
4

Hyperthyroidism
5

Lead poisoning
Q/Q(M)-474583 Report a Problem

Plummer's nails are associated with which of the following disorders?
4

Hyperthyroidism
Plummer's nails are onycholysis due to hyperthyroidism.
Q/Q(M)-474583 Report a Problem


Cutaneous signs of cortisol excess include all of the following EXCEPT:
1

Plethora
2

Atrophic/fragile skin
3

Striae distensae
4

Dorsocervical/Supraclavicular fat pads
5

Peripheral obesity
Q/Q(M)-478084 Report a Problem


Cutaneous signs of cortisol excess include all of the following EXCEPT:
5

Peripheral obesity
All of the listed options are signs of cortisol excess except peripheral obesity. The most common pattern
of obesity in patients with cortisol excess is centripetal or central obesity.
Q/Q(M)-478084 Report a Problem


Koilonychia is most commonly seen in association with:
1

Renal disease
2

Progressive systemic sclerosis
3

Alopecia areata
4

Impaired iron metabolism
5

Pulmonary disease
Q/Q(M)-476961 Report a Problem


Koilonychia is most commonly seen in association with:
4

Impaired iron metabolism
In koilonychia (spoon nails), the nail is a concave shape with raised lateral edges. Iron-deficiency anemia
66

and Plummer-Vinson syndrome have been associated with koilonychia. Involvement of only the first three
fingernails is suggestive of an occupational cause. In early infancy, koilonychia is a normal finding.
Koilonychia is also seen in hemochromatosis, an autosomal recessive condition associated with increased
intestinal iron absorption and increased iron deposition leading to metallic-grey hyperpigmentation of the
skin, hepatomegaly, diabetes, hypogonadism, polyarthritis, and cardiac abnormalities.
Q/Q(M)-476961 Report a Problem

A patient with a history of gastrointestinal polyposis presents with alopecia, generalized pigmentation, and
nail dystrophy. Which of the following is the most likely diagnosis?
1

Peutz-Jegher syndrome
2

Gardner syndrome
3

Cronkhite-Canada Syndrome
4

Cowden syndrome
5

Dyskeratosis congenita
Q/Q(M)-474556 Report a Problem


A patient with a history of gastrointestinal polyposis presents with alopecia, generalized pigmentation, and
nail dystrophy. Which of the following is the most likely diagnosis?
3

Cronkhite-Canada Syndrome
Cronkhite-Canada Syndrome is a sporadic gastrointestinal polyposis syndrome characterized by alopecia,
generalized pigmentation, and nail dystrophy.
Q/Q(M)-474556 Report a Problem


A patient with synophrys, dystopia canthorum, and heterochromia iridum irides likely has which of the
following hair abnormalities?
1

Diffuse thinning
2

Scarring alopecia
3

Trichorrhexis invaginata
4

Trichorrhexis nodosa
5

White forelock
Q/Q(M)-474564 Report a Problem



A patient with synophrys, dystopia canthorum, and heterochromia iridum irides likely has which of the
following hair abnormalities?
5

White forelock
White forelock is a common feature of Waardenburg syndrome.
Q/Q(M)-474564 Report a Problem



67

A 52-year-old man presents to clinic with erythematous papules and pustules in the region of the cheeks,
chin, and neck that are worsened by shaving. Examination reveals pseudofolliculitis barbae. With what
disease may this be associated:
1

Cirrhosis
2

Heart Failure
3

Chronic Renal Failure
4

Myeloma
5

Emphysema
Q/Q(M)-482940 Report a Problem


A 52-year-old man presents to clinic with erythematous papules and pustules in the region of the cheeks,
chin, and neck that are worsened by shaving. Examination reveals pseudofolliculitis barbae. With what
disease may this be associated:
3

Chronic Renal Failure
Pseudofolliculitis barbae has been associated with renal failure requiring hemodialysis, particularly in
Caucasian patients. Most commonly, this syndrome arises in patients with darker skin types, whose hair
shafts are elliptical in cross section, rather than round. This elliptical nature predisposes the hairs to
kinking and invagination into the skin.
Q/Q(M)-482940 Report a Problem

S-ar putea să vă placă și